CSIR-UGC-NET Previous Placement Paper - Life Sciences

CSIR-UGC-NET Previous Placement Paper - Life Sciences 1. Popular use of which of the following fertilizers increases the acidity of soil? (1) Potassium Nitrate (2) Ammonium sulphate (3) Urea (4) Superphosphate of lime 2. Exposing an organism to a certain chemical can change nucleotide bases in a gene, causing mutation. In one such mutated organism if a protein had only 70% of the primary ammo acid sequence, which of the following is likely? (1) Mutation broke the protein (2) The organism could not make amino acids (3) Mutation created a terminator codon (4)The gene was not transcribed 3. If the atmospheric concentration of carbon di oxide is doubled and there are favourable conditions of water, nutrients, light and temperature, what would happen to water requirement of plants? (1) It decreases initially for short time and then return to original value (2) It increases (3) It decreases (4) It increases initially for short time and then return to original value 4. A physiological disorder X always leads to the disorder Y. However, disorder Y may occur by itself. A population shows 4% incidence of disorder Y. Which of the following inferences is valid? (1) 4% of the population suffers from both X & Y (2) Less than 4% of the population suffers from X (3) At least 4% of the population suffers from X (4) There is no incidence of X in the given population 5. Diabetic patients are advised a low glycaemic index diet. The reason for this is (1) They require less carbohydrate than healthy individuals (2) They cannot assimilate ordinary carbohydrates (3) They need to have slow, but sustained release of glucose in their blood stream (4) They can tolerate lower, but not higher than normal blood sugar levels 6. The reason for the hardness of diamond is (1) extended covalent bonding (2) layered structure (3) formation of cage structures (4) formation of tubular structures 7. Which of the following particles has the largest range in a given medium if their initial energies are the same? (1) alpha (2) gamma (3) positron (4) electron 8. Standing on a polished stone floor one feels colder than on a rough floor of the same stone. This is because (1) Thermal conductivity of the stone depends on the surface smoothness (2) Specific heat of the stone changes by polishing it (3) The temperature of the polished floor is lower than that of the rough floor (4) There is greater heat loss from the soles of the feet when in contact with the polished floor than with the rough floor 9. Recent studies on Archaea suggest that life could have originated (1) extraterrestrially and seeded through meteorite impacts. (2) in shallow coastal areas. (3) in deep hydrothermal vents.. (4) in hot, terrestrial habitats. 10. If the ratio of the number of nonsynonymous to snynonymous substitutions per site in protein coding gene is greater than one, it is an evidence of selection that is (1) positive. (2) negative. (3) neutral. (4) random. 11. The frequencies of alleles 'A' and 'a' in a population at Hardy- Weinburg equilibrium are 0.7 and 0.3, respectively. In a random sample of250 individuals taken from the population, how many are expected to be heterozygous? (1) 112 (2) 81 (3) 105 (4) 145 12. The transition to flowering in plants requires (1) growth of plants under long- day conditions. (2) growth of plants under short- day conditions. (3) reprogramming of the shoot apical meristem. (4) synthesis of the flowering hormone florigen 13. Aneuploid females with only one X chromosome is a characteristic of individuals with (1) Cri du chat syndrome (2) Klinefelter syndrome (3) Down syndrome. (4) Turner syndrome. 14. Which of the following food crops has recently been genetically engineered to obtain edible vaccine to develop immunity against hepatitis B? (1)Banana (2) Maize (3) Potato (4) Tomato 15. The most commonly used method of estimating primary productivity of a pond involves measurement of the amount (1) CO2 utilized (2) autotroph biomass. (3) oxygen released. (4) organic carbon. 16. A much greater proportion of energy fixed by autotrophs is transferred to the herbivore level in the open ocean ecosystem than in a forest ecosystem because (1)aquatic autotrophs are small. (2)aquatic herbivores are more efficient feeders. (3) terrestrial autotrophs are less efficient feeders. (4) terrestrial autotrophs have more indigestible tissues. 17. Which of the cyclins have/has essential functions in S-phase of cell cycle? (1) A-type. (2) B-type. (3) D-type. (4) Both B- and D-types. 18. A mechanism that can cause a gene to move from one linkage group to another is (1) crossing over. (2) inversion. (3) translocation. (4) duplication. 19. Cytoplasmic determinants coding for anterior structure of Drosophila embryo if injected elsewhere in the recipient embryo, would lead to (1)normal development. (2) formation of additional ectopic head. (3) degeneration. (4) a phenotype with two heads and two tails. 20. ELISA assay uses (1) an enzyme which can react with secondary antibody. (2) an enzyme which can react with the antigen. (3) a substrate which gets converted into a coloured product. (4) a radiolabelled secondary antibody 21. The 5' Cap of RNA is required for the (1) stability of RNA only. (2) stability and transport of RNA. (3) transport of RNA only. (4) methylation of RNA. 22. In amphibian oocyte, the germplasm which gets segregated during cleavage to give rise to primordial germ cells (PGC's) is normally (1)distributed evenly throughout the oocyte. (2) localized at animal pole. (3) localized at vegetal pole. (4) aggregated in central part of oocyte. 23. In mature Arabidopsis embryo, root apical meristem consists of cells derived from (1) embryo and apical suspensor cells (2) embryo only (3) suspensor only. (4) hypophysis only. 24. A mother of blood group 0 has a group a child. The father could be of blood type (1) A or B or O. (2) A only. (3) A or B. (4) AB only. 25. Name the ectothermic animal that can thermoregulate by behavioural means rather than by physiological means. (1) Bumble bee in an orchard. (2) Tuna fish in the ocean. (3) Lizard in a desert. (4) Flatworm in a pond. 26. Which of the following methods of plant transformation can be used to introduce a gene into chloroplast genome? (1) Agrobacterium-mediated transformation (2) Particle delivery system (3)Permeabilization (4)Electroporation 27. Which of the following characteristic of an early community? (1) Narrow niche specialization. (2) High species diversity. (3) community production. (4) open mineral cycling. 28. During urine formation the filtration of blood at the glomerulus is (1) an active process. (2) an osmotic process. (3) is a pressure-dependent physical process. (4) a non energy- mediated transport process. 29. With which protein of Yersinia would integrin proteins of mammalian cells interact for internalization? (1). Pilin (2) Fimbrin, (3) lnvasin (4) Adherin 30. Graves disease is associated with (1) insufficiency of thyroid hormones. (2) excess of thyroid hormones. (3) insufficiency of corticosteroids. (4) excess of growth hormones. CSIR-UGC-NET Placement Paper CSIR Model Question Paper Earth Sciences Model Question Paper: 1. Albedo of a surface is defined by the ratio of outgoing to incoming solar radiation. Keeping this in view, which of the following surface will have the highest albedo? 1. Water 2. Sand 3. Snow 4. Forest 2. Geostrophic wind is a balance between the pressure gradient force and the Coriolis force. In non-geostrophic wind, flow will become circulatory because of 1. pressure difference term 2. momentum term 3. temperature difference term 4. shear term 3. In valley-mountain system, cold air is heavier than warm air, and thus flows down hill as a katabatic wind. In the case of an anabatic wind, 1. wind rises vertically in the valley 2. wind forms updraft and downdraft motion 3. wind will flow along the slope 4. wind will have cross mountain flow 4. Cyclones or hurricanes do not develop within about 50 N and S of the equator because the 1. pressure gradient is weak 2. trade winds converge 3. ocean surface temperature is high 4. Coriolis force is very weak 5. When statically stable air flown over a hill or ridge, it is set into oscillation. These waves are known as 1. Rossby waves 2. Kelvin waves 3. gravity waves 4. mountain waves 6. A ceaseless movement of wind and ocean currents results in global circulation because of 1. unequal distribution of oceanic and continental mass 2. radiative forcing due to Sun-Earth interaction 3. tilt of Earth's axis 4. revolution / rotation of Earth 7. The organism that can take N2 directly from sea water is 1. foraminifera 2. cocolithophore 3. cyanobacteria 4. dinoflagellates 8. Which is the major limiting element in marine primary production? 1. sodium 2. nitrogen 3. carbon 4. argon 9. If the sea surface temperature is increased in Antarctic Ocean due to global warming, the dissolved oxygen in its deep water will 1. increase 2. initially increase and then decrease 3. decrease 4. not change 10. Hydrogenous sediment in the oceans is 1. derived from skeletal debris 2. precipitated by chemical or biochemical reactions 3. produced by weathering of rocks on land 4. ejected by volcanoes 11. Sea surface temperature (SST) is an important and variable parameter of oceans.In the Indian Ocean, SST variability has the largest amplitude on the timescale of 1. century 2. decadal 3. interannual 4. seasonal 12. The speed of current in the wind-driven Ekman layer 1. increases with depth 2. decreases with depth 3. remains constant 4. first decreases and then increases 13. Which one of the following is not true in case of the Arabian Sea? 1. seasonal high biological productivity 2. mid-depth oxygen minimum zone 3. net annual evaporation 4. net annual precipitation 14. Silicon limitation in the upper ocean will hamper the growth of 1. cocolithophos 2. foraminifera 3. diatam 4. corals 15. Which one of the following rock suites represents the oceanic lithosphere? 1. turbidite 2. ophiolite 3. seismite 4. granulite 16. Which one of the following is defined by shear strain? 1. change in length 2. change in angle 3. change in volume 4. change in area 17. Which one of the following is a measure of compressibility of a material? 1. elasticity (E) Young's modulus 2. rigidity (G) shear modulus 3. Poisson's ratio (?) 4. viscosity (?) 18. A light beam is incident on a mirror at an angle q with the normal to the mirror. If the mirror is rotated in the direction of incident beam by an angle r, what will be the rotation angle of the reflected beam with respect to its previous position? 1. r 2. r + q 3. r - q 4. 2r 19. Coesite, a high pressure polymorph of quartz is found in rocks exposed on Earth's surface because, coesite 1. is stable at low temperature 2. is a high density mineral 3. is not kinetically favoured for decomposition 4. has the lowest free energy at the Earth's surface 20. In Al2SiO5, the Al2O3 content (in wt %) is close to 1. 80 2. 60 3. 40 4. 20 21. A region with nearly flat topography and large negative gravity anomaly suggests 1. a root in the mantle 2. thin crust with low density material underplating 3. thick crust with high density material underplating 4. thin crust 22. Generation of nuclear energy requires availability of minerals containing U and Th. Which one of the following rocks is known to have high concentrations of nuclear minerals? 1. basalt 2. diorite 3. granodiorite 4. pegmatite 23. Which one of the following minerals will have no useful nutrient elements for plant life? 1. plagioclase feldspar 2. quartz 3. olivine 4. biotite 24. The atmosphere of Venus has about 97% CO2. If so, the planet should have 1. an abundance of photosynthesizing bacteria 2. an abundance of water 3. no life at all as the surface should be very hot 4. an abundance of fossil fuel deposits 25. Which one of the following drainage patterns is typically associated with fold mountain belts? 1. parallel 2. rectangular 3. trellis 4. dendritic 26. If the wave front is exactly parallel to the coast, which one of the following will still occur along the coast? 1. beach drift 2. longshore drift 3. longshore current 4. wave refraction 27. A river will braid irrespective of the fact that it is gravel-bed, sand-bed or silt-bed if 1. sediment supply = sediment transport 2. sediment supply < sediment transport 3. sediment supply > sediment transport 4. sediment supply >> sediment transport 28. If R is the ratio of the Earth?s gravity field to its magnetic field at the equator, the ratio of these fields at the poles would be approximately 1. R/4 2. R/2 3. R 4. 2R 29. On the geoidal surface, the Earth?s gravity field 1. is everywhere zero 2. is a constant and is normal to the surface 3. varies along the surface, but is normal to it 4. varies in magnitude and direction along the surface 30. An isostatically compensated elevated landmass is characterized by 1. strong negative Bouguer and free-air anomalies 2. strong negative Bouguer and little or no free-air anomalies 3. strong positive Bouguer and little or no free-air anomalies 4. strong positive Bouguer and free-air anomalies 31. The remnant magnetism in a rock solidified in the Earth?s magnetic field at alatitude of 30°, dips 1. at 30° 2. between 30°and 45° 3. between 45° and 60° 4. at an angle greater than 60° 32. The shadow zone is the region bounded by the disappearance of P and S waves,and the onset of P waves 1. reflected at the Earth?s surface 2. refracted through the fluid core 3. reflected at the mantle-core boundary 4. refracted through the fluid and solid cores 33. Statement I: Ferromagnetic minerals exhibit properties of paramagnetic minerals beyond Curie temperatures. Statement II: Mantle rocks are less magnetic than those of the upper crust. 1. statements I and II are true; I explains II 2. statements I and II are true, but I does not explain II 3. statement I is true, but II is false 4. statement I is false, but II is true 34. An example of intraplate volcanic activity is 1. the Himalaya 2. island arcs 3. the Alps 4. the Hawaiian islands CSIR-UGC-NET Placement Paper CSIR Exam Preparation - Study Material General Science Paper: 1. Which of the following bonds will be most difficult to break? 1. C-O 2. C-C 3. C-N 4. C-S 2. The conformation of a nucleotide in DNA is affected by rotation about how many bonds? 1. 4 2. 6 3. 7 4. 3 3. Which of the following proteins acts as an energy transducer? 1. G-protein. 2. Bacteriorhodopsin. 3. Hemoglobin. 4. Heat shock protein. 4. Which of the following predicted property of lipid bilayers would result if the phospholipids had only one hydrocarbon chain instead of two? 1. The bilayers formed would be much less fluid. 2. The diameter of the head group would be much larger than the acyl chain and would tend to form micelles rather than bilayers. 3. the bilayers formed would be much more fluid. 4. the bilayers would be more permeable to small water-soluble molecules. 5. Which of the following statements is not true for transposable element system? 1. It consists of both autonomous and non-autonomous elements. 2. Dissociation elements are autonomous in nature. 3. Transposase is transcribed by the central region of autonomous elements. 4. Certain repeats in the genome remain fixed even after the element transposes out. 6. A set of virulence genes (vir genes), located in the Agrobacterium Ti-plasmid, is activated by 1. octopine. 2. nopaline. 3. acetosyringone. 4. auxin 7. When two mutants having the same phenotype were crossed, the progeny obtained showed a wild-type phenotype. Thus the mutations are 1. non-allelic. 2. allelic. 3. segregating from each other. 4. independently assorting. 8. Two varieties of maize averaging 48 and 72 inches in height, respectively, are crossed. The F1 progeny is quite uniform averaging 60 inches in height. Of the 500 F2 plants, the shortest 2 are 48 inches and the tallest 2 are 72 inches. What is the probable number of polygenes involved in this trait? 1. Four. 2. Eight. 3. Sixteen. 4. Thirty two. 9. Repair of double strand breaks made during meiosis in the yeast Saccharomyces cerevisiae 1. occurs mostly by non-homologous end joining. 2. occurs mostly using the sister chromatid as a template. 3. occurs mostly using the homologous chromosome as a template. 4. is associated with a high frequency of mutations. 10. Which of the following signaling molecules enters the cell to initiate its action? 1. Transferrin 2. Insulin 3. Glucagon 4. Thyroxine 11. The mode of action of the anticancer drug methotrexate is through its strong competitive inhibition on 1. dihydrofolate reductase. 2. thymidine synthase. 3. thymidine kinase. 4. adenylate cyclase. 12. Which class of immunoglobulins will increase in case of a chronic infection? 1. IgA 2. IgG 3. IgM 4. IgE 13. When prospective neuroectoderm from an early amphibian gastrula is transplanted in the prospective epidermal region of a recipient (early gastrula) embryo, the donor tissue will give rise to 1. neural tube. 2. epidermis. 3. neural tube and notochord. 4. neural tube and epidermis. 14. Amphibian oocytes remain for years in the diplotene stage of meiotic prophase. Resumption of meiosis is initiated by 1. gonodatropic hormone. 2. growth hormone. 3. oestrogen. 4. progesterone. 15. A group of six cells called 'equivalence group cells' divide to form the vulval structure in Caenorhabditis elegans. They are called so because 1. they have similar fates during development of vulva. 2. all the six cells are competent to form vulva and can replace each other under various experimental conditions. 3. they are all under the influence of the anchor cell, signals from which initiate vulval development. 4. they interact with each other to form the vulval structure. 16. Due to the presence of cellulose in the cell wall of plants, leaf shape is determined in the leaf primorida by 1. rates of cell division. 2. planes of cell division. 3. cell migration. 4. cell-cell interactions. 17. DCMU inhibits electron transport in chloroplast by preventing the reduction of 1. P 680. 2. QA. 3. PQ. 4. QB. 18. In higher plant leaves, the reduction of nitrate to ammonium takes place by the combined action of nitrate reductase localized in cytosol and nitrite reductase localized in 1. peroxisomes. 2. mitochondria. 3. chloroplasts. 4. cytosol. 19. In higher plants, the red/far-red sensory photoreceptor, phytochrome, is a light-regulated kinase. Which of the following classes of kinases does it represent? 1. Two-component sensor regulator (histidine kinase). 2. Two-component sensor regulator (serine/threonine kinase). 3. Leucine rich repeat (LRR) receptor kinase. 4. Calcium-dependent protein kinase. 20. Vesicular-arbuscular mycorrhiza (VAM) represents a beneficial association between plant roots and fungus, where fungus assists plants in obtaining from the soil 1. iron. 2. zinc. 3. sulphate. 4. phosphate. 21. Unidirectional propagation of electrical signal in nervous system is 1. proportional to the length of axon. 2. due to chemical synapse. 3. due to electrical synapse. 4. proportional to myelination. 22. A myasthenia gravis patient develops muscle paralysis because 1. the nerve terminal at the neuromuscular junction fails to release acetylcholine. 2. although enough acetylcholine is released at the neuromuscular junction, it is destroyed by acetylcholinesterase. 3. the patient develops immunity against his own acetylcholine receptor. 4. the patient develops antibody against his own acetylcholine. 23. Inhibin from sertoli cells of testes selectively inhibits 1. luteinizing hormone. 2. follicle stimulating hormone. 3. thyroid stimulating hormone. 4. growth hormone. 24. Hawk's retina possesses a large number of 1. rods. 2. melanocytes. 3. cones. 4. kuffer cells. 25. Which of the following plant groups evolved during the Silurian period? 1. Bryophyta 2. Psilophyta 3. Lycophyta 4. Spherrophyta 26. Which of the following processes does not take place in the 5??3? direction? 1. DNA replication 2. Transcription 3. Nick translation 4. RNA editing 27. A deletion of three consecutive bases in the coding region of a gene cannot result in 1. deletion of a single amino acid without any other change in the protein. 2. replacement of two adjacent amino acids by a single amino acid. 3. replacement of a single amino acid by another without any other change in sequence of the protein. 4. production of a truncated protein. 28. Deletion of the leader sequence of trp operon of E. coli would result in 1. decreased transcription of trp operon. 2. increased transcription of trp operon. 3. no effect on transcription. 4. decreased transcription of trp operon in the presence of tryptophan. 29. In the endodermis of higher plants, the role of Casperian strip is to control the water movement so that it flows 1. between the cells. 2. through the plasma membrane. 3. through the cell wall. 4. through the transfusion tissue. 30. The reptilian order Squamata includes 1. crocodiles and alligators. 2. the living fossil 'tuatara'. 3. turtles and tortoises. 4. snakes and lizards. 31. Cultivated bananas are sterile because 1. male flower-bearing plants are very rare. 2. they lack natural pollinators in the crop plants. 3. they are triploid and therefore seeds are not set. 4. they are a cross of two unrelated species. 32. One life history trait that is not characteristic of very small sized organisms is 1. delayed age at first reproduction. 2. earlier age at first reproduction. 3. high population growth rate. 4. short lifespan. 33. Which of the following statements is the most appropriate example of character displacement? 1. Two related species depending on the same prey species avoid competition by feeding at different times of the day. 2. The body sizes of two related species are very similar when they are allopatric, but in geographical areas of sympatry, one species is significantly smaller than the other. 3. The food niche of a species is generally wider in the absence of competing species than in their presence. 4. Closely related species can coexist if their densities are regulated by a predator. 34. Evolutionarily, with which of the following could parental care in animals be associated? 1. Smaller clutch size. 2. Polygamy. 3. Greater longevity. 4. Semelparity. 35. During which geological period did the greatest diversification of life on earth occur? 1. Permian 2. Jurassic 3. Cambrian 4. Ordovician CSIR Exam Preparation - Study Material General Science Paper: 1. Which of the following bonds will be most difficult to break? 1. C-O 2. C-C 3. C-N 4. C-S 2. The conformation of a nucleotide in DNA is affected by rotation about how many bonds? 1. 4 2. 6 3. 7 4. 3 3. Which of the following proteins acts as an energy transducer? 1. G-protein. 2. Bacteriorhodopsin. 3. Hemoglobin. 4. Heat shock protein. 4. Which of the following predicted property of lipid bilayers would result if the phospholipids had only one hydrocarbon chain instead of two? 1. The bilayers formed would be much less fluid. 2. The diameter of the head group would be much larger than the acyl chain and would tend to form micelles rather than bilayers. 3. the bilayers formed would be much more fluid. 4. the bilayers would be more permeable to small water-soluble molecules. 5. Which of the following statements is not true for transposable element system? 1. It consists of both autonomous and non-autonomous elements. 2. Dissociation elements are autonomous in nature. 3. Transposase is transcribed by the central region of autonomous elements. 4. Certain repeats in the genome remain fixed even after the element transposes out. 6. A set of virulence genes (vir genes), located in the Agrobacterium Ti-plasmid, is activated by 1. octopine. 2. nopaline. 3. acetosyringone. 4. auxin 7. When two mutants having the same phenotype were crossed, the progeny obtained showed a wild-type phenotype. Thus the mutations are 1. non-allelic. 2. allelic. 3. segregating from each other. 4. independently assorting. 8. Two varieties of maize averaging 48 and 72 inches in height, respectively, are crossed. The F1 progeny is quite uniform averaging 60 inches in height. Of the 500 F2 plants, the shortest 2 are 48 inches and the tallest 2 are 72 inches. What is the probable number of polygenes involved in this trait? 1. Four. 2. Eight. 3. Sixteen. 4. Thirty two. 9. Repair of double strand breaks made during meiosis in the yeast Saccharomyces cerevisiae 1. occurs mostly by non-homologous end joining. 2. occurs mostly using the sister chromatid as a template. 3. occurs mostly using the homologous chromosome as a template. 4. is associated with a high frequency of mutations. 10. Which of the following signaling molecules enters the cell to initiate its action? 1. Transferrin 2. Insulin 3. Glucagon 4. Thyroxine 11. The mode of action of the anticancer drug methotrexate is through its strong competitive inhibition on 1. dihydrofolate reductase. 2. thymidine synthase. 3. thymidine kinase. 4. adenylate cyclase. 12. Which class of immunoglobulins will increase in case of a chronic infection? 1. IgA 2. IgG 3. IgM 4. IgE 13. When prospective neuroectoderm from an early amphibian gastrula is transplanted in the prospective epidermal region of a recipient (early gastrula) embryo, the donor tissue will give rise to 1. neural tube. 2. epidermis. 3. neural tube and notochord. 4. neural tube and epidermis. 14. Amphibian oocytes remain for years in the diplotene stage of meiotic prophase. Resumption of meiosis is initiated by 1. gonodatropic hormone. 2. growth hormone. 3. oestrogen. 4. progesterone. 15. A group of six cells called 'equivalence group cells' divide to form the vulval structure in Caenorhabditis elegans. They are called so because 1. they have similar fates during development of vulva. 2. all the six cells are competent to form vulva and can replace each other under various experimental conditions. 3. they are all under the influence of the anchor cell, signals from which initiate vulval development. 4. they interact with each other to form the vulval structure. 16. Due to the presence of cellulose in the cell wall of plants, leaf shape is determined in the leaf primorida by 1. rates of cell division. 2. planes of cell division. 3. cell migration. 4. cell-cell interactions. 17. DCMU inhibits electron transport in chloroplast by preventing the reduction of 1. P 680. 2. QA. 3. PQ. 4. QB. 18. In higher plant leaves, the reduction of nitrate to ammonium takes place by the combined action of nitrate reductase localized in cytosol and nitrite reductase localized in 1. peroxisomes. 2. mitochondria. 3. chloroplasts. 4. cytosol. 19. In higher plants, the red/far-red sensory photoreceptor, phytochrome, is a light-regulated kinase. Which of the following classes of kinases does it represent? 1. Two-component sensor regulator (histidine kinase). 2. Two-component sensor regulator (serine/threonine kinase). 3. Leucine rich repeat (LRR) receptor kinase. 4. Calcium-dependent protein kinase. 20. Vesicular-arbuscular mycorrhiza (VAM) represents a beneficial association between plant roots and fungus, where fungus assists plants in obtaining from the soil 1. iron. 2. zinc. 3. sulphate. 4. phosphate. 21. Unidirectional propagation of electrical signal in nervous system is 1. proportional to the length of axon. 2. due to chemical synapse. 3. due to electrical synapse. 4. proportional to myelination. 22. A myasthenia gravis patient develops muscle paralysis because 1. the nerve terminal at the neuromuscular junction fails to release acetylcholine. 2. although enough acetylcholine is released at the neuromuscular junction, it is destroyed by acetylcholinesterase. 3. the patient develops immunity against his own acetylcholine receptor. 4. the patient develops antibody against his own acetylcholine. 23. Inhibin from sertoli cells of testes selectively inhibits 1. luteinizing hormone. 2. follicle stimulating hormone. 3. thyroid stimulating hormone. 4. growth hormone. 24. Hawk's retina possesses a large number of 1. rods. 2. melanocytes. 3. cones. 4. kuffer cells. 25. Which of the following plant groups evolved during the Silurian period? 1. Bryophyta 2. Psilophyta 3. Lycophyta 4. Spherrophyta 26. Which of the following processes does not take place in the 5??3? direction? 1. DNA replication 2. Transcription 3. Nick translation 4. RNA editing 27. A deletion of three consecutive bases in the coding region of a gene cannot result in 1. deletion of a single amino acid without any other change in the protein. 2. replacement of two adjacent amino acids by a single amino acid. 3. replacement of a single amino acid by another without any other change in sequence of the protein. 4. production of a truncated protein. 28. Deletion of the leader sequence of trp operon of E. coli would result in 1. decreased transcription of trp operon. 2. increased transcription of trp operon. 3. no effect on transcription. 4. decreased transcription of trp operon in the presence of tryptophan. 29. In the endodermis of higher plants, the role of Casperian strip is to control the water movement so that it flows 1. between the cells. 2. through the plasma membrane. 3. through the cell wall. 4. through the transfusion tissue. 30. The reptilian order Squamata includes 1. crocodiles and alligators. 2. the living fossil 'tuatara'. 3. turtles and tortoises. 4. snakes and lizards. 31. Cultivated bananas are sterile because 1. male flower-bearing plants are very rare. 2. they lack natural pollinators in the crop plants. 3. they are triploid and therefore seeds are not set. 4. they are a cross of two unrelated species. 32. One life history trait that is not characteristic of very small sized organisms is 1. delayed age at first reproduction. 2. earlier age at first reproduction. 3. high population growth rate. 4. short lifespan. 33. Which of the following statements is the most appropriate example of character displacement? 1. Two related species depending on the same prey species avoid competition by feeding at different times of the day. 2. The body sizes of two related species are very similar when they are allopatric, but in geographical areas of sympatry, one species is significantly smaller than the other. 3. The food niche of a species is generally wider in the absence of competing species than in their presence. 4. Closely related species can coexist if their densities are regulated by a predator. 34. Evolutionarily, with which of the following could parental care in animals be associated? 1. Smaller clutch size. 2. Polygamy. 3. Greater longevity. 4. Semelparity. 35. During which geological period did the greatest diversification of life on earth occur? 1. Permian 2. Jurassic 3. Cambrian 4. Ordovician Placement Paper CSIR UGC NET -2013Question Bank in Computer Science Programming Languages and C Multiple Choice Questions, CSIR UGC NET 2013 Engineering science,Mathematical science, Life Science etc...,Written test pattern Examination procedure, CSIR UGC NET 2013 previous years solved question papers, CSIR UGC NET model questions for practice 1. Electronic spreadsheets are most useful in a situation where relatively ............. data must be input, and but ........ calculations are required (a) little; simple (b) large; simple (c) large; complex (d) little; complex (Ans) 2. The two basic types of record-access method are: (a) sequential and random (Ans) (b) direct and immediate (c) sequential and indexed (d) on — line and real — time (e) None of the above 3. Which file organization is allowed by a direct-access storage device ? (a) direct only (b) sequential and direct only (c) indexed and direct only (d) sequential, indexed and direct (Ans) (e) None of the above 4. Sequential file organization is most appropriate for which of following applications ? (a) Grocery-store checkout (b) Bank checking account (c) Payroll (Ans) (d) Airline reservations (e) None of the above 5. Which of the following file organization is most efficient for a file with a high degree of file activity ? (a) Sequential (Ans) (b) ISAM (c) VSAM (d) B-Tree index (e) All of the above 6. One disadvantage of a direct access file is : (a) the delay in computing the storage address (Ans) (b) duplication of address locations (c) unused, but available, storage locations (d) All of the above. 7. All computers execute (a) BASIC programs (b) COBOL programs (c) Machine language programs (Ans) (d) FORTRAN programs (e) PL/1 programs 8. Which of the following is most oriented to scientific programming ? (a) FORTRAN (Ans) (b) COBOL (c) BASIC (d) PL/1 (e) RPG 9. All of the following are disadvantages of RPG except : (a) it is a very machine-dependent language (b) it is very limited in its scope (c) is not suited for complex problems requiring extensive programming logic (d) it has larger storage requirements (Ans) (e) All of the above are disadvantages 10. Which of the following is not one of the processes that a high-level language program must go through before it is ready to be executed ? (a) translation (b) controlling (Ans) (c) lading (d) linking (e) All of the above are necessary processes 11. Which of the following is not true of FORTRAN ? (a) it was developed for scientific and mathematical applications (b) it is one of the oldest high-level languages (c) it is a problem-oriented language (Ans) (d) it requires extensive internal documentation (e) All of the above are true 12. All of the following are divisions of the COBOL program except : (a) Input-output (Ans) (b) Identification (c) Procedure (d) Data (e) All of the above are divisions 13. In a COBOL program, the input-output section is within the .............. division. (a) Identification (b) procedure (c) configuration (d) environment (Ans) (e) None of the above 14. Which of the following is not a characteristic of COBOL ? (a) it is a very standardized language (b) it is a very efficient in terms of coding and execution (Ans) (c) it has limited facilities for mathematical notation (d) it is very readable language (e) All of the above are characteristics 15. Which of the following is an example of problem-oriented language ? (a) BASIC (b) PL/1 (c) FORTRAN (d) All of the above (e) None of the above (Ans) 16. In the evaluation of a computer language, all of the following characteristics should be considered except ? (a) application-oriented features (b) efficiency (c) readability (d) software development aids (e) hardware maintenance costs (Ans) 17. A factor in the selection of a source language is (a) programmer skill (b) language availability (c) program compatibility with other software (d) All of the above (Ans) 18. A computer-generated output that lets programmers follow the execution of their programs line by line is a (a) core dump (b) tracing routine (Ans) (c) detail listing (d) source listing 19. In BASIC, descriptive comments are put in the source program with the (a) PRINT statement (b) REMARK statement (Ans) (c) INPUT statement (d) DATA statement 20. Which of the following generations of languages will likely include the languages of the feature ? (a) First generation (b) Second generation (c) third generation (d) Fourth generation (e) Fifth generation (Ans) 21. Which of the following use (s) English-like statements to represent program logic ? (a) Flow charts (b) ANSI symbols (c) Pseudocode (Ans) (d) Algorithm (e) None of the above 22. Which of the following generation does natural language fall into ? (a) first generation (b) second generation (c) third generation (d) fourth generation (e) fifth generation (Ans) 23. Which of the following relates to machine language ? (a) difficult to learn (b) first-generation language (c) machine-independent (d) instructions and data are represented by binary digits (e) All of the above (Ans) 24. Which of the following might prevent a program from being modified in the future ? (a) logic errors (b) pseudocode (c) lack of program documentation (Ans) (d) syntax errors (e) All of the above 25. Which of the following is/are used to speed up the process of designing the initial version of a software program ? (a) ANSI (b) BASIC (c) Compiler (d) Applications generator (Ans) (e) All of the above 26. Which of the following helped develop standards for high-level programming languages ? (a) ISO (b) IBM (c) CCITT (d) AT & T (e) ANSI (Ans) 27. The language that does not require the user to learn a specific vocabulary of syntax is : (a) RPG (Ans) (b) FORTRAN (c) COBOL (d) BASIC (e) None of the above 28. Which of the following is a high-level language ? (a) Ada (b) BASIC (c) COBOL (d) FORTRAN (e) All of the above (Ans) 29. Which of the following allows users to specify what the computer is supposed to do rather than how the computer is supposed to do it ? (a) non procedural language (Ans) (b) FORTRAN (c) BASIC (d) COBOL (e) None of the above 30. A computer program consists of (a) a completed flowchart (b) algorithms (c) algorithms written in computer's language (Ans) (d) discrete logic steps 31. The use of a computer to get information from a data bank is called (a) information withdrawal (b) computer simulation (c) information retrieval (Ans) (d) full-text searching 32. The person who writes instructions that tell the computer how to handle input information is called ? (a) data entry check (b) computer programmer (Ans) (c) computer operator (d) keypunch operator 33. A computer programmer (a) does all the thinking for a computer (Ans) (b) can enter input data quickly (c) can operate all types of computer equipment (d) can draw only flowcharts 34. PILOT language is meant for (a) tailors (b) school-going children (c) authors, teachers, and trainers (Ans) (d) use in process control 35. The special-purpose computer language for process control is (a) ADA (Ans) (b) COMAL (c) PL/1 (d) PASCAL 36. Low level computer language use (a) English words (b) Mnemonic codes (Ans) (c) Limited grammar (d) Mathematical symbols 37. A source program is the program written in ......... language (a) English (b) Symbolic (c) High-level (Ans) (d) Machine 38. A program written in machine language is called ............ (a) Program (b) Assembler (Ans) (c) object (d) computer (e) machine 39. Instructions for execution by a computer are given in ........... language. (a) symbolic (b) BASIC (c) Machine (Ans) (d) Compiler 40. Mark the wrong statement (a) Digital computers work on sequential basis (b) Programs written in high-level languages are know as source programs (c) Every computer has its own machine language (d) An assembler is needed to translate assembly language program into machine (e) High-level languages are easy to learn and use but are less efficient than machine (f) A given compiler can be used with practically all modern computers (Ans) 41. In computer terminology a compiler means (a) a person who compiles source program (b) the same thing as a programmer (c) keypunch operator (d) a program which translates source program into object program (Ans) 42. In reference to a computer, an assembles is a (a) program (Ans) (b) person who assembles the parts (c) symbol (d) language 43. Relate the following terms a. BASIC b. FORTRAN c. COBOL d. APPLE e. PASCAL f. PILOT i. Beginner's Language ii. Teacher's Language iii. Business Language iv. Personal computer v. Scientific language vi. Structured language (a) a-I, b-ii, c-iii, d-iv, e-v, f-vi (b) a-I, b-ii, c-iii, d-iv, e-v, f-iv (c) a-I, b-v, c-iii, d-iv, e-vi, f-ii (Ans) (d) a-v, b-vi, c-iv, d-iii, e-ii, f-I 44. Which of the following language is not well suited for business applications ? (a) COBOL (b) PL/1 (c) Assembly (Ans) 45. Which of the following language is usually implemented with an interpreter ? (a) Assembly (b) PASCAL (c) COBOL (d) BASIC (Ans) 46. Which of the following language is often translated to pesudo code ? (a) Assembly (b) FORTRAN (c) PL/1 (d) BASIC (e) PASCAL (Ans) 47. Can you name of the most primitive computer programming language ? (a) machine languages (Ans) (b) assembly language (c) 4GL (d) high level language 48. An instruction that can be recognized and used without translation must be written in (a) BASIC (b) Machine code (Ans) (c) Assembly code (d) Source code 49. Compilers ad interpreters are themselves (a) high level languages (b) programs (Ans) (c) codes (d) mnemonics 50. What is the name of the output form wither of compiler or an assembler ? (a) Source code (b) Object code (Ans) (c) Op-coded (d) Operand CSIR Question papers CSIR UGC NET -2013Question Bank in Computer Science PAPER ? I solved question papers for learn and practice 1. Feedback in communication Process __________________ (A) Explores defect in receiver (B) Clarifies the communication (Ans) (C) Beneficial in understanding (D) Delights the Sender Hints and Solution : Feedback is the return of a portion of the output of a process or system. Feedback is used to maintain, control, and clarify the performance. 2. Intrapersonal communication is __________________ (A) Person-to-person contact (B) Talking to oneself (Ans) (C) When more than two persons are involved (D) None of the above Hints and Solution : Intrapersonal communication is language used internal to the communicator. It occurs in the mind of the individual in a model, which contains a sender, receiver, and feedback loop. 3. An effective communication produces in receiver ______ (A) Acceptance (Ans) (B) Joy (C) Thinking (D) None of the above Hints and Solution : An effective communication produces in receiver acceptance. Questions 4-8 : Study the following graph carefully to answer the given questions. Production of Two Companies A and B (In Crore Units) Over The Given Years 4. For Company A, how much is the percent increase in production in 2000 from 1999? (A) 0.25 (B) 2.5 (C) 25 (Ans) (D)12.5 Hints and Solution : % increase = 0.25crore × 100 1 crore = 25 % 5. How many units is the total production of Company A for the given years? (A) 9 crores (B) 17.75 crores (C) 12.25 crores (D) 11 crores (Ans) Hints and Solution : for company A 1 + 1.25 + 1.5 +1.75 + 1.75 + 1.75 + 2 = 11 crore 6. What is the difference in units produced by the two companies in 1999? (A) 1,50,000,000 (Ans) (B) 15,00,00,000 (C) 15,00,000 (D) 15,000 Hints and Solution : (2.5 ? 1) crores = 1.5 × 100 00 000 = 150 00 000 7. How many units is the approximate average production of Company B for the given years? (A) 3 crores (B) 2.55 crores (Ans) (C) 2.75 crores (D) 2.25 crores Hints and Solution : Total production of B = 2.5 + 2.5 + 3 + 2.75 +2.25 + 2.25 +2.5 = 17.75 Average production = 17.75 7 = 2.55 8. In which year did both the companies have no change in production from the previous year? (A) 2000 (B) 2002 (C) 2003 (D) 2004 (Ans) Hints and Solution : In year 2004. 9 Which of the following is not open source software? (A) Internet Explorer (Ans) (B) Apache HTTP Server (C) Fedora Linux (D) Open Office Hints and Solution : Internet Explorer is not open source software. 10. In MS Word, after selecting content, it can be copied by ______ (A) Shift Key + C Key (B) Alt Key + C Key (C) Enter Key (D) Ctrl Key + C Key (Ans) Hints and Solution : In MS Word, after selecting a content, it can be copied by Ctrl Key + C Key. 11 MODEM = ______________ (A) An Electric Device (B) Modulator and Demodulator (Ans) (C) Internal Memory (D) None of the above Hints and Solution : MODEM = Modulator and Demodulator 12 Multimedia is __________ (A) A computer Game (B) A software (C) A technology (Ans) (D) A Media Hints and Solution : Multimedia is a technology 13 What is the full form of ?LAN?? (A) Large Area Network (B) Local Area Network (Ans) (C) Little Area Network (D) None of the above Hints and Solution : The full form LAN is Local Area Network. 14 Look at this series: 53, 53, 40, 40, 27, 27, what number should come next? (A) 10 (B) 18 (C) 14 (Ans) (D) 45 Hints and Solution : The series follow According to the series the next number would be 14. Direction (15-19) The strength of Indian Democracy lies in its tradition, in the fusion of the ideas of democracy and national independence which was the characteristic of the Indian Nationalist Movement long before independence. Although the British retained supreme authority in India until 1947, the provincial elections of 1937 provided real exercise in democratic practice before national independence. During the Pacific war India was not overrun or seriously invaded by the Japanese and after the war was over, the transfer of power to a government of the Indian Congress Party was a peaceful one as far as Britain was concerned. By 1947 'Indianisation' had already gone far in the Indian Civil Service and Army, so that the new government could start with effective instruments of central control. After independence, however, India was faced with two vast problems; the first, that of economic growth from a very low level of production and the second was that of ethnic diversity and the aspirations of sub nationalities. The Congress leadership was more aware of the former problem than of the second. As a new political elite which had rebelled not only against the British Raj but also against India's old social order, they were conscious of the need to initiate economic development and undertake social reforms, but as nationalists who had led a struggle against the alien rule on behalf of all parts of India, they took the cohesion of the Indian nation too much for granted and underestimated the centrifugal forces of ethnic division, which were bound to be accentuated rather than diminished as the popular masses were more and more drawn into politics. The Congress party was originally opposed to the idea of recognizing any division of India on a linguistic basis and preferred to retain the old provinces of British India which often cut across linguistic boundaries. However, this was later conceded as the basis for a federal 'Indian Union'. The rights granted to the States created new problems for the Central Government. The idea of making Hindi the national language of a united India was thwarted by the recalcitrance of the speakers of other important Indian languages and the autonomy of the States rendered central economic planning extremely difficult. Land reforms remained under the control of the States and many large-scale economic projects required a degree of cooperation between the Central Government and one or more of the States which, it was found, was impossible to achieve. Coordination of policies was difficult even when the Congress party was in power both in the State and at the Centre. When a Congress Government in Delhi was confronted with non-Congress parties in office in the States, it became much harder. 15. Which of the following problems was India faced with after Independence ? (A) Military attack from a country across the border. (B) Lack of coordination between the Central and State Governments. (C) Improper coordination of various Government policies (D) Increasing the production from a very low level (Ans) Hints and Solution : Production was at very low level. 16. Which of the following issues was not appropriately realized by the Central Government. (A) Ethnic diversity of the people (Ans) (B)A national language for the country (C) Implementation of the formulated policies (D) Centre -State relations Hints and Solution : Ethnic diversity of the people was not appropriately realized by the Central Government. 17. Why was central economic planning found to be difficult? (A) Multiplicity of States and Union Territories (B) Lack of coordination in different Government departments (C) Autonomy given to the States in certain matters (Ans) (D) Lack of will in implementing land reforms Hints and Solution : Central economic planning found to be difficult because autonomy was given to the States in certain matters 18. Why was the linguistic reorganization of the State accepted? (A) The States were not cooperating with the Central Government (B) Non- Congress Governments in the States demanded such a reorganization of the States (C) No common national language emerged (D) Strong pressure from the States was exerted on the Central Government to create such States (Ans) Hints and Solution : Because that time no common language emerged. 19. Which, according to the passage, can be cited as an exercise in democratic practice in India before Independence? (A) The handing over of power by the British to India (B) The Indianisation of the Indian Civil Service (Ans) (C) A neutral role played by the Army (D) None of the above Hints and Solution : ?The Indianisation of the Indian Civil Service? , can be cited as an exercise in democratic practice in India before Independence 20. The information to be collected in survey method are related to (A) Present Position (B) Aims of the research (C) The attainment of aim of research (D) All of the above (Ans) Hints and Solution : The information to be collected in survey method are related to present position, aims of the research &the attainment of aim of research 21. Research is done for (A) Knowledge of research process (B) Solving a business problem (Ans) (C) Interest in research (D) Experience Hints and Solution : Research is done for solving a business problem 22 A research problem is feasible only when (A) It is researchable (B) It has some utility (C) It is new (D) All of the above (Ans) Hints and Solution : A research problem is feasible only when it is researchable, it has some utility, and it is new. 23. One of the essential characteristics of research is (A) Sensitivity (B) Generalizability (C) Usability (Ans) (D) Replicability Hints and Solution : One of the essential characteristics of research is usability. 24. The experimental research is (A) A method for verifying a hypothesis (Ans) (B) A variable controlling method (C) A method of deriving inferences (D) None of the above Hints and Solution : This is an experiment where the researcher manipulates one variable & controls the rest of the variables. So it is a method for verifying a hypothesis. 25. UGC was established formally as statutory body in the year ____________ (A) 1972 (B) 1965 (C) 1956 (Ans) (D) 1962 Hints and Solution : UGC was established formally as statutory body in the year 1956. 26. In India, total numbers of Open Universities are _______ (A) Fourteen (Ans) (B) Sixteen (C) Eighteen (D) Fifteen Hints and Solution : In India, total numbers of Open Universities are Fourteen. 27. Who was chairman of the UGC committee 1969 appointed for administrative legislation of the universities? (A). Dr. Zakir Hussain (B) Dr. P B Gajendragadkar (Ans) (C) Dr. L S Mudaliar (D) Dr. Radha Krishnan Hints and Solution : Dr. P B Gajendragadkar was chairman of the UGC committee 1969 appointed for administrative legislation of the universities. 28. UGC has launched career oriented program in (A) 1964-65 (B) 1994-95 (Ans) (C) 1997-98 (D) 1980-81 Hints and Solution : UGC has launched career oriented program in 1994-95. 29. The prime minister of India is appointed from ______ (A) The leading Party in Lok Sabha (Ans) (B) The Leading Party in Rajya Sabha (C) The leading party in Lok Sabha and Rajya Sabha combined (D) None of the above Hints and Solution : The prime minister of India is appointed from the leading Party in Lok Sabha. 30. The study of interrelations between Organism and their environment is called___ (A) Biosphere (B) Ecology (Ans) (C) Synecology (D) Autecology Hints and Solution : The study of interrelations between Organism and their environment is called ecology. 31 WWF is _______ (A) World wide Fund for People (B) World welfare Fund (C) World wide Fund for Nature (Ans) (D) Non of the above Hints and Solution : WWF is world wide Fund for Nature. 32 Which was the first National Park established in India? (A) Anshi National Park (B) Gir National Park (C) Kanha National Park (D) Jim Corbett National Park (Ans) Hints and Solution : Jim Corbett National Park was the first National Park established in India. 33 Fossil Fuels include (A) Oil (B) Natural Gas (C) Coal (D) All of the above (Ans) Hints and Solution : Fossil fuels are fuels made by natural processes such as anaerobic decomposition of buried dead organisms. Ex. Oil, Natural gas, coal etc. 34 Noise in excess of ______ is called noise pollution (A) 40-65 db (B) 60-70 db (C) 80-100 db (Ans) (D) None of the above Hints and Solution : Noise in excess of 80-100DB is called noise pollution. 35 Effectiveness of teaching depends on ____ (A) Handwriting of Teacher (B) Speaking ability of Teacher (C) Qualification of the Teacher (D) Subject Understanding of the Teacher (Ans) Hints and Solution : Effectiveness of teaching depends on Subject Understanding of the Teacher 36 Verbal Guidance is least effective in the learning of ____ (A) Aptitudes (B) Skills (C) Attitudes (D) Relationship (Ans) Hints and Solution : Verbal Guidance is least effective in the learning of Relationship. 37 The participation of students will be maximum if ____ method is used for teaching. (A) Text Books (B) Discussion Method (Ans) (C) Conference Method (D) Lectures Hints and Solution : The participation of students will be maximum if Discussion Method is used for teaching. 38 The primary responsibility of the teacher?s adjustment lies with (A) The Students (B) The Principal (C) The Community (D) The Teacher himself (Ans) Hints and Solution : The primary responsibility of the teacher?s adjustment lies with The Teacher himself 39. The First Kindergarten was started by (A) William James (B) A D Clinton (C) Freidrich Forebel (Ans) (D) J H Hills Hints and Solution : The First Kindergarten was started by Freidrich Forebel 40. In following questions, number series is given. One of the numbers in each series is wrong. After searching wrong number find the correct number in its place. 510, 254, 126, 64, 30, 14, 6 (A) 252 (B) 62 (Ans) (C) 130 (D) 9 Hints and Solution : 30, 64 is wrong & must be replaced by 62. 41. Cow : Calf :: Mother: ? (A) Daughter (Ans) (B) Father (C) Brother (D) None of the above Hints and Solution : Cow : Calf :: Mother : Daughter 42. Insert the missing number or letter from among the given alternatives (A) 140 (B) 280 (C) 875 (D) 925 (Ans) Hints and Solution : (2)2 = 4, (5)2 = 25 ? 425 (2)2 = 4, (4)2 = 16 ? 416 (3)2 = 9, (5)2 = 25 ? 925. 43. In the following question assuming the given statements to be true, find out which of the two assumptions I and II given below them is/are definitely true give answer as. (A) Only assumption I is implicit (Ans) (B) Only assumption II is implicit (C) Either I or II is implicit (D) Neither I nor II is implicit (E) Both I and II are implicit Hints and Solution : Such decisions as given in the statement are taken only after taking the existing vacancies into consideration. So, I implicit while II does not implicit. Statement : The State government has decided to appoint four thousand primary school teachers during the next financial year. Assumptions : I. There are enough schools in the state to accommodate four thousand additional primary school teachers. II. The eligible candidates may not be interested to apply as the government may not finally appoint such a large number of primary school teachers. 44. What is the latest write-once optical storage media? (A) Digital paper (B) Magneto-optical disk (C) WORM disk (D) CD-ROM disk (Ans) Hints and Solution : CD-ROM disk is the latest write-once optical storage media 45. Which of the following identifies a specific web page and its computer on the Web? (A) Web site (B) Web site address (C) URL (D) Domain Name (Ans) Hints and Solution : Domain Name identifies a specific web page and its computer on the Web. Direction (46-47) In the following figure, rectangle, square, circle and triangle represents the regions of wheat gram, maize and rice cultivation respectively. On the basis of the figure, answer the following questions. 46. Which of the area is cultivated for wheat and maize only? (A) 8 (B) 6 (C) 5 (D) 4 (Ans) Hints and Solution : The required region is the one which is common only to the rectangle and circle and is not a part of either the triangle or square 47. Which of the area is cultivated for maize only? (A) 10 (B) 2 (C) 3 (Ans) (D) 4 Hints and Solution : The required region is the one which lies inside the circle but outside the rectangle, square and triangle, 48. Pointing to a photograph. Bajpai said, ?He is the son of the only daughter of the father of my brother.? How Bajpai is related to the man in the photograph? (A) Nephew (B) Brother (C) Father (D) Maternal Uncle (Ans) Hints and Solution : The man in the photo is the son of the sister of Bajpai. Hence, Bajpai is the maternal uncle of the man in the photograph. 49. Light Year is a unit of: (A) Intensity of light (B) Distance (Ans) (C) Time (D) Planetary motion Hints and Solution : Light Year is a unit of distance. 50. Tsunamis are huge sea waves caused by : (A) Earthquakes (Ans) (B) Volcanoes (C) Winds (D) Icebergs Hints and Solution : Tsunamis are huge sea waves caused by earthquakes UGC Question Paper-English UGC NET English questions,UGC NET Teaching aptitude questions Directions: Read the passage carefully and answer the question No-1 -5 A hundred years ago there was much less specialisation in work than there is today. One furniture maker would make the whole of a table- indeed, perhaps, all the furniture needed to furnish a house, and he worked in a simple workshop, probably entirely with hand tools. The Furniture he made was very good and very beautiful, but it would take him a longtime to make it. The output of one man was, by modern standards, small, and the cost of the Furniture relatively high. Only the fairly wealthy could afford a great deal. Most modern furniture is made in fatories with the aid of machinery. So now-a-days a man working in a furniture factory will, perhaps spend his days mindling one machine which carries out one process only in the making of table legs. A great number of men will be taking part in the various process which go to making the table, and by this division of labour a great many tables will be made in a comparatively short time. A good workman however should understand all the process which go to make the table, even though he himself actually does only one of them. In the making of more complicated things, such as cars or elaborate pieces of electrical machinery, even this is not possible. It is very likely even that all the processes do not go on in the same factory. 1. ‘Mindling’ in the passage can he replaced by (a) Thinking of (b) Feeling (c) Taking care of (d) Touching Ans:-C 2. Why was the cost of the furniture so high in the past? (a) Output was small (b) Material was costly (c) Artisans were greedy (d) None of the above Ans:-A 3. Production of Furniture is quicker because (a) Of division of labour (b) Of machinery (c) Good conditions of working (d) Of technology development Ans:-A 4. The artisans of the ancient times did not have (a) Skill (c) Markets (b) Specialisation (d) (a) and (c) both Ans:-B 5. Which of the following uses of ‘even’ are correct? (a) The chances of success are even (b) Even a child can do it (c) The cost roated in the even (d) (b) and (c) both Ans:-A 6. In the following alphabet which letter is seventh to the right of fifteenth letter from the right end? A B C D E F G H I J K L M N O PQ R S T U V W X Y X Z (a) S (b) T (c) R (d) H (e) None of these Ans:-A 7. Two letters in the word ROUBLE have as many letters between them in the word as in the alphabet. Which one of the two comes earlier in the alphabet? (a) L (b) O (c) E (d) R (e) B Ans:-E 8. What should come in place of the question mark? In the following letter series? AYD BVF DRH ? KGL (a) FMI (b) GMJ (c) HLK (d) GLJ (e) None of these Ans:-B 9. Which among the following are the right measures to solve the obstacles in relation to women education? (a) To increase girl schools (b) To encourage and motivate people (c) To solve economic problems for the people (d) All of these Ans:-D 10. Which commission recommended General School education to be of 10 years? (a) Mudaliar education commission (b) Kothari commission (c) Hunter commission (d) All the above Ans:-B 11. Which of the following are elements of education planning? (a) Problems (b) Builders (c) Resources (d) All the above Ans:-D 12. Which method would you deem suitable for rectifying those students who interrupt the teaching process in the class? (a) They should be sent out of the class (b) They should be given harsh punishment in the class itself (c) They should be ignored (d) A complaint should be filed against them with the principal Ans:-A 13. As a teacher, what type of behaviour of a student would be deemed serious by you? (a) Incomplete home work (b) Asking too many questions in the class (c) Talking to other students in the class during the course of a lecture (d) Slightly slow/weak in studies Ans:-C 14. Abhinav was born 2 years after his Fathers marriage. His mother is Five year younger than his Father but. 20 years older than Abhinav. At what age did his Father get married? (a) 35 years (c) 23 years (b) 33 years (d) 25 years Ans:-C 15. If 84 x 13 = 8, 37 ×13=6, 26 x 11=6, then 56 x 22=? (a) 36 (b) 39 (c) 7 (d) 11 Ans:-C 16. Bibliography given in a research report (a) Helps those interested in further research and studying the problem from another angle (b) Makes the report authentic (c) Shows the vast knowledge of the researchers (d) None of the above Ans:-A 17. The experimental study is based on the law of (a) Single variable (b) Replication (c) Occupation (d) Interest of the Subject Ans:-A 18. How can the objectivity of research be enhanced? (a) Through its impartiality (b) Through its reliability (c) Through its validity (d) All of the above Ans:-D 19. Starting from a point P Satish walked 20 metres towards south. He turned left and walked 30 metres. He then turned left and walked 20 metres. He again turned left and walked 40 metres and reached a point Q. How far and in which direction is the point Q from the point P? (a) 20 metres West (b) 10 metres East (c) 10 minutes West (d) 10 metres North (e) None of these. Ans:-C 20. Mukesh is taller than Rajeev but shorter than Amar. Suman is taller than Mukesh but shorter than Vinay. Who among them will be at the Fourth place if they stand in a row in descending order of their height? (a) Amar (b) Rajeev (c) Vinay (d) Cannot be determined. (e) None of these Ans:-E 21. Which of the following is determiner of education? (a) Home (b) School (c) Family (d) All the above Ans:-D 22. The main aim of Basic Education is (a) To be self-dependent and self-sufficient (b) To depend on the teacher (c) To depend on the society (d) None of these Ans:-A 23. The job of the teacher is to generate: (a) Affiancing for studies among students (b) Confidence in studies among students (c) A belief towards studies among students (d) Affect/affiliation for studies among students. Ans:-B 24. In the profession of teaching, the most beneficial aspect is (a)An opportunity to dominate the children (b)An opportunity to express yourself (c)Hefty earnings from tution work, besides earnings through salary (d)Sufficient number of holidays in a year. Ans:-B 25. Survey study aims at I. Knowing Facts about the existing situation the chief social II.Comparing the present status with the standard norms. III. Criticising the existing situation 1V. Identifying the means of improving the existing situation (a) I and II only (b) I, II and III (c) I, II, and III and IV (d) II and III only Ans:-B 26. Logic of introduction is very close to (a) The logic of sampling (b) The logic of observation (c) The logic of the controlled variable (d) None of the above Ans:-A CSIR Exam Paper - Life Sciences 1. In India, brown antlered deer (sangai) is found only in the floating landmasses of (1) Wular lake. (2) Sasthamkotta lake. (3) Dal lake. (4) Lok Tak lake 2. Both halophytes and glycophytes compartmentalize cytotoxic ions into the intracellular compartment or actively pump them out of the cell to the apoplasts with the help of membrane transport proteins. Among these, the Na+-H+ antiporter, NHX1, is localized in (1) the plasma membrane. (2) chloroplast (inner envelope). (3) mitochondria (outer membrane). (4) Tonoplast. 3. You are studying the binding of proteins to the cytoplasmic face of cultured liver cells and have found a method that gives a good yield of inside-out vesicles from the plasma membrane. Unfortunately, your preparations are contaminated with variable amounts of right-side-out vesicles. Nothing you have tried avoids this contamination. Somebody suggests that you pass the vesicles over an affinity column made of lectin coupled to Sepharose beads. What is the rational of this suggestion? (1) Right-side-out-vesicles wIll be lysed by lectin coupled to Sepharose beads. (2) Right-side-out-vesicles will simply bind to the lectin coupled Sepharose beads. (3) Lectin will bind to the carbohydrate residues present only on the inside out vesicles. (4) Lectin will bind to only glycoproteins and glycolipids present on the inside-out vesicles. 4. Eukaryotic genomes are organized into chromosomes and can be visualized at mitosis by staining with specific dyes. Heat denaturation followed by staining with Giemsa produced alternate dark and light bands. The dark bands obtained by this process are mainly (1) AT -rich and gene rich regions. (2) AT -rich and gene desert regions. (3) GC-rich and gene rich regions. (4) GC-rich and gene desert regions. 5. Lac repressor inhibits expression of genes in lac-operon whereas purine biosynthesis is repressed by the Pur repressor. The two proteins have 31% identical sequences and have similar three-dimensional structures. The gene regulatory properties of these proteins differ in relation to A) binding of small molecules to the repressor. B) presence of recognition sites on the genome. C) oligomeric nature of the repressor. D) DNA binding property. The correct statements are (1) A and B (2) A, B and C (3) A and C (4) B, C and D 6. Two E. coli cultures A and B are taken. Culture A was earlier grown in the presence of optimum concentration of gratuitous inducer IPTG. Both the cultures are now used to inoculate fresh medium containing sub-optimal concentration of gratuitous inducer. It was observed that culture B was unable to utilize lactose, whereas culture A did so efficiently. The reason behind this is (1) pretreatment with IPTG has resulted in a mutation as a result of which lac operon is constitutively expressed (2) IPTG has made the cell membrane more porous to small molecu1es and so lactose is taken up more efficiently by A as compared to B. (3) in culture A, lactose permease was induced to a high level, during pretreatment with IPTG, which allowed the preferential uptake of lactose. (4) in culture A, IPTG activated a receptor which bound lactose more efficiently, thereby triggering a signal. 7. Cancer causing genes can be functionally classified into mainly three types: (i) genes that induce cellular proliferation, (ii) tumor suppressor genes, (iii) genes that regulate apoptotic pathway. Epstein-Barr virus that causes cancer by modulating apoptotic pathway, contains a gene having sequence homology with which of the following genes? (1) bax (2) bcl-2 (3) p53 (4) caspase-3 8. It has been observed that in 5-10% of the eukaryotic mRNAs with multiple AUGs, the first AUG is not the initiation site. In such cases, the ribosome skips over one or more AUGs before encountering the favourable one and initiating translation. This is postulated to be due to the presence of the following consensus sequence (s): A) CCA CC AUG G B) CCG CC AUG G C) CCG CC AUG C D) AAC GG AUG A Which of the following sequence sets related to the above postu1ations is correct? (1) A and B (2) A and C (3) C and D (4) B and D 9. Presence of circular mRNAs for a specific protein in an eukaryotic cell reflects a rapid rate of synthesis of that protein. Following mechanisms are suggested: A) eIF-4G and PABP promote this process through 5'-3' interaction of mRNA. B) ribosomes are less active in recognizing circular mRNA. C) PABP and eIF-4A promote this process. D) ribosomes can reinitiate translation without being disassembled. Which of the following is correct? (1) A and D (2) B and D (3) A and C (4) B and C 10. siRNAs and miRNAs are used for achieving gene silencing. Although, major steps are similar there are distinct differences in the key players of the two processing pathways. Following statements relate to some characteristic features of gene silencing. A) Both siRNAs and miRNAs are processed by cytoplasmic endonuclease Dicer. B) 'Drosha' is needed for processing miRNAs and precursor siRNAs. C) Both siRNAs and miRNAs show association with Argonaute protein. D) Both the processing pathways involve RISe complex. Which of the following combinations is NOT correct? (1) A and C (2) C and D (3) A and B (4) D and A 11. Glucose is mobilized in muscle when epinephrine activates G?s. In an experiment in which muscle cells were stimulated with epinephrine, glucose mobilization was observed even after withdrawal of epinephrine. This could be (1) due to the presence of a cAMP phosphodiesterase inhibitor. (2) very low rates of cyclic AMP formation. (3) due to the presence of a cAMP phosphodiesterase activator. (4) due to the absence of protein kinase A. 12. In eukaryotic chromatin, 30 nm fiber (solenoid) can open up to give rise to two kinds of chromatin. In one type (A), the promoter of a gene within the open chromatin is occupied by a nucleosome whereas in the other (B), the promoter is occupied by histone H1. The following possibilities are suggested. A) The gene in (A) is repressed. B) The gene in (B) is repressed. C) The gene in (A) is active. D) The gene in (B) is active. Which of the following sets is correct? (1)A and D (2) A and B (3) Band D (4) C and D 13. Following are some statements for synthesis of secondary metabolites in plants. A) Terpenes are synthesized by shikimic acid pathway and mevalonic acid pathway. B) Alkaloids are nitrogen containing compounds and are synthesized by shikimic acid pathway. C) Phenolic compounds are synthesized by shikimic acid pathway and mevalonic acid pathway. D) Both alkaloids and terpenes are synthesized by mevalonic acid pathway and MEP pathway. Which one of the following combinations of the above statement is true? (1) A and D (2) A and C (3) B and C (4) B and D 14. During early cleavage of Caenorabditis elegans embryos, each asymmetrical division produces one founder cell which produces differentiated descendants and one stem cell. The very first cell division produces one anterior founder cell, namely AB and one posterior stem cell, namely P1. When these blastomeres are experimentally separated and allowed to proceed further with development, one could get the following possible outcomes: (1) P1 cell would develop autonomously while the AB would show conditional development. (2) P1 cells would show conditional development while AB would show autonomous development. (3) Both would show autonomous specification and result in mosaic development. (4) Both would show conditional specification and result in regulative development. 15. Two new plant species, A and B, were described in 1872. Subsequently it was found that the type for species A was never designated and for species B there was one specimen designated as type but missing. As per International Code of Botanical Nomenclature (ICBN), typification should be (1) neotype for A only. (2) neotype for A and lectotype for B. (3) neotypes for both A and B. (4) lectotype for both A and B. 16. According to MacArthur and Wilson's equilibrium theory, which of the following is true? (1) Larger islands and islands closer to continent are expected to have more species than smaller and isolated islands. (2) Smaller islands and islands far from the continent are expected to have more species than larger and isolated islands. (3) Smaller islands and islands closer to the continent are expected to have more species than far away smaller and isolated islands. (4) More species are expected on all islands irrespective of their size and distance from the continent. 17. Gause's 'Competitive exclusion' principle states that two species with identical niches cannot coexist indefinitely. Which of the following statements is the most appropriate regarding the validity of the principle? (1) It depends on how one defines niche. (2) There are in nature many instances of continued coexistence of closely related species. (3) The principle is universally true. (4) It does not predict the outcome where, both the species are equally strong competitors. 18. In a lake ecosystem, bottom-up effects (B) refers to control of a lower trophic level by the higher trophic levels and top down effects (T) refer to the opposite. In a lake with three trophic levels - Phytoplankton ( P), Zooplankton (Z) and Carnivore (C), (1)P and C are controlled by B, and Z is controlled by T (2) P, Z and C are all controlled by T (3) P is controlled by B, Z is controlled by T and C is controlled by B (4) P is controlled by T, Z is controlled by B and C is controlled by T 19. What will be the approximate effective population size in a panmictic population of 240 with 200 females and 40 polygamous males? (1) 160 (2) 133 (3) 63 (4) 67 20. An animal was first maintained in a constant environmental condition for several days until a consistent biological rhythm (B) was established. The animal was then exposed to an experimental physical rhythm (E).whichmodulates the phase and period of B. However, upon withdrawal of E, the B gradually regained its pattern of pre-exposure condition. From these observations which one or more of the following should be the most logical inference? A) E is a Zeitgeber. B) E is a masking agent. C) E causes entrainment of B. D) B is a conditioned to E. The correct answer is (1) A and C (2) B and D (3) B only (4) D only 21. What is the minimum number of NTPs required for the formation of one peptide bond during protein synthesis? (1) One (2) Two (3) Four (4) Six 22. 'Imperfect fungi' is a group represented by fungal species which have (1) simple mycelia. (2) no known mechanisms of sexual reproduction. (3) unkown phylogenetic relationship. (4) lost its survival mechanism against harsh environments. 23. The photoreceptor commonly involved In light entrainment of the biological clock in flies, moulds and plants is (1) phytochrome. (2) rhodopsin (3) carotenoid. (4) Cryptochrome. 24. The free energy ?G of a dissolved solute (1) Increases with solute concentration. (2) decreases with solute concentration (3) is independent of solute concentration. (4) depends only on temperature. 25. Which of the following is not a characteristic of phylum Chordata? (1)Pharyngeal slits (2) Amniotic egg (3) Post anal tail (4) Notochord CSIR English Question Papers CSIR English Language Type of Questions: Objective (Multiple Choice) Paper I: PART - A ENGLISH LANGUAGE Directions: In Question Nos. 1 to 10, some of the sentences have errors and some have none. Find out which part of a sentence has an error and choose the option corresponding to the appropriate letter (A, B, C). If there is no error mark (D) as your answer. 1. A moment delay/would have proved costly/ in the situation./No error. 2. Ram disappointed his mother/ as he did not/ write to her very often./No error. 3. After you will return/ from Chennai/ I will come and see you./ No error. 4. Scenes from the film / had to be censured/ before it was released./ No error. 5. When my sister was ill/ I went to the hospital/ on alternative days./ No error. 6 The beautiful/ surrounding of the place/ enchanted me./ No error. 7. No Porter being available/ he carried/ all his luggages himself./ No error. 8 He will not be able/ to cope up with/ the pressure of work./ No error. 9. Lasers are/ indispensable tools/ for the delicate eyes surgery./ No error. 10. I take great pleasure/ to welcome you/ to this institution./ No error. Directions: In Question Nos. 11 to 15, out of the four alternatives, choose the one which expresses the right meaning of the given. 11. Indignation (A) hatred (B) anger (C) disapproval (D) contempt 12. Acronym (A) A word with two or more meanings (B) A word of new coinage (C) A word formed by the initial letters of words (D) A word of picturesque effect 13. Meticulous (A) interfere (B) courage (C) agreement (D) careful 14. Rescind (A) change (B) revoke (C) repeat (D) reconsider 15. Antipathy (A) dishonesty (B) disturbance (C) demonstration (D) dislike Directions: In Question Nos. 16 to 20, choose the word opposite in meaning to the given word. 16. APPARENT (A) Illegible (B) Hidden (C) Mysterious (D) Remote 17. ALIEN (A) Native (B) Domiciled (C) Natural (D) Resident 18. FUTILE (A) upright (B) costly (C) eminent (D) worthy 19. AUDACIOUS (A) meek (B) cowardly (C) mild (D) gentle 20. ARROGANT (A) simple (B) timid (C) civilized (D) modest Directions: In Question Nos. 21 to 30, sentences are given with blanks to be filled in with an appropriate and suitable word. 21. The food that an average Indian eats has been found to be deficient _____ vitamins and proteins. (A) of (B) from (C) with (D) in 22. The patient is _____ of stomachache. (A) suffering (B) experiencing (C) complaining (D) afflicting 23. A good teacher should _____ responses from the students. (A) elicit (B) provoke (C) command (D) infer 24. Had he taken his degree five years ago he _____ got a promotion by now. (A) might (B) would have (C) will be (D) was 25. There will be a rush for seats when the train (A) will arrive (B) arrived (C) is arriving (D) arrives 26. He drove the car very fast _____ (A) Did he? (B) Does he? (C) Didn't he? (D) Was he? 27. He _____ wants to succeed in life must be prepared to work hard. (A) whoever (B) whom (C) who (D) whose 28. It is time you _____ home. (A) go (B) went (C) come (D) reach 29. _____ a walk in the morning will improve your health. (A) Going to (B) Go to (C) Go for (D) Going for 30. Government must _____ the rise in prices. (A) cheque (B) check (C) cease (D) seize Directions: In Question Nos. 31 to 40, four alternatives are given for the underlined or given idiom/phrase. Choose the alternative which best expresses the meaning of the underlined or given idiom/phrase. 31. The bus had a close shave as its driver swerved to the right a split second before the on-coming truck could run into it. (A) serious accident (B) close collision (C) narrow escape (D) deep dent 32. Fits and starts (A) slowly (B) not regularly (C) continuously (D) quickly 33. When the Inspector entered the class some of the students shook in their shoes. (A) stamped the ground with their shoes (B) showed signs of anger (C) trembled with fear (D) stood up to salute 34. In high spirits (A) full of hope and enthusiasm (B) under tremendous stress (C) under the influence of liquor (D) mentally deranged 35. He amassed his wealth through sharp practices. (A) dishonest means (B) illegal means (C) intelligent decisions (D) quick decisions 36. He is not in the good books of his boss. (A) a lover of good books (B) in favour with (C) not of the same opinion as (D) as good as 37. The officer is fed up with the complaints made against the clerk. (A) annoyed (B) disgusted (C) pleased (D) satisfied 38. A white elephant, (A) a rare species of elephants (B) an expensive gift (C) a costly but useless possession (D) a worthless thing 39. Ins and outs (A) entry and exit points (B) full details (C) tactical moves (D) complexity of character 40. All his ventures went to the winds. (A) dissipated (B) spread all over (C) got speed of the winds (D) became well-known Directions: In Question Nos. 41 to 50, out of the four alternatives, choose the one which can be substituted for the given words/sentence. 41. Government by a ruler who has unlimited power. (A) Despotism (B) Autocracy (C) Monarchy (D) Anarchy 42. An occasion of great importance (A) exemplary (B) momentous (C) herculean (D) grandiose 43. A person who is always hopeful and look: upon the brighter side of things (A) florist (B) artist (C) theist (D) optimist 44. Place of burial (A) cave (B) church (C) synagogue (D) cemetry 45. To have a very high opinion of oneself (A) exaggeration (B) adulation (C) abundance (D) conceited 46. One who believes in giving equal opportunity to women in all fields (A) Fanatic (B) Misogynist (C) Philanderer (D) Feminist 47. Inability to sleep (A) hysteria (B) insomnia (C) aphasia (D) amnesia 48. One who is given to pleasures of the flesh. (A) terrestrian (B) epicurean (C) celestial (D) pedestrian 49. A tank where fish or water plants are kept (A) Aquarium (B) Sanatorium (C) Nursery (D) Aviary 50. A person who never takes alcoholic drinks (A) teetotaller (B) alcoholic (C) addict (D) bagpiper Directions: In Question Nos. 51 to 55, four words are given in each question, out of which only one word is wrongly spelt. 51. (A) poignant (B) relevent (C) prevalent (D) malignant 52. (A) seize (B) achieve (C) wierd (D) leisure 53. (A) repent (B) serpent (C) flagrent (D) reverent 54. (A) dining (B) shining (C) determining (D) begining 55. (A) vendetta ' (B) verisimilitude (C) vicarious (D) vociferrate Directions: In Question Nos. 56 to 65, a part of the sentence is underlined. Below are given alternatives to the underlined part at A, B and C which may improve the sentence. Choose the correct alternative. In case no improvement is needed, your answer is D. 56. He was weary of failure, Didn't he? (A) Isn't he? (B) Wasn't he? (C) Doesn't he? (D) No improvement 57. What you have been doing since the work* shop last month? (A) have you done (B) you have done (C) have you been doing (D) No improvement 58. Keats says the idea very well in his poems. (A) speaks (B) describes (C) expresses (D) No improvement 59. The enemy soldiers went back hastily. (A) returned (B) retreated (C) retrenched (D) No improvement 60. No sooner had the teacher entered the room and the boys rushed to their seats. (A) when (B) than (C) but (D) No improvement 61. We have already disposed our old house. (A) disposed off (B) disposed out (C) disposed of (D) No improvement 62. We can buy anything in this shop, Can we? (A) Isn't it? (B) Can't we? (C) Don't we? (D) No improvement 63. If I had followed your advice, I would not regret today. (A) will not regret (B) had not regretted (C) would not have regretted (D) No improvement 64. Somebody must be made to answer for the securities scam. (A) to (B) after (C) upon (D) No improvement 65. Corruption is the most serious problem in India. (A) the more serious (B) the seriouser (C) serious (D) No improvement Directions: In Question Nos. 66 to 70, a sentence has been given in Active/Passive Voice. Out of the four alternatives suggested below, select the one which best expresses the same sentence in Passive/Active Voice. 66. The cat is running after the rat. (A) The rat was being run after by the cat (B) The rat is being run after by the cat (C) The rat is run after by the cat (D) The cat is being run after by the rat 67. English is spoken all over the world. (A) All over the world English speaks (B) English speaks all over the world (C) The whole world speaks English (D) People speak English all over the world 68. The boys elected Mohan captain. (A) The boys were elected captain by Mohan (B) Mohan is elected captain by the boys (C) Mohan was elected captain by the boys (D) Mohan and the boys elected the captain 69. They threw away the Rubbish. (A) The Rubbish will be thrown away (B) The Rubbish was being thrown away (C) The Rubbish was thrown away (D) The Rubbish thrown away 70. Let him see the picture. (A) Let the picture be seen by him (B) The picture is seen by him (C) Let him the picture be seen (D) The picture is seen by him Directions: In Question Nos. 71 to 80, the first and the last parts of the sentence are numbered 1 and 6. The rest of the sentence is split into four Parts and named P, Q, R and S. These four parts are not given in their proper order. Read the parts and find out which of the four combinations s correct. Then find the correct answer. 71. 1. There was a lamp hanging on the wall P. Then he picked a lot of jewels off the trees in the garden Q. He put the lamp inside his shirt R. He put them in his pockets S. Aladdin climbed a ladder and took down the lamp 6. Then he put more jewels inside his shirt on top of the lamp. (A) PRQS (B) RSPQ (C) SQPR (D) QSRP 72. 1. Gopal and Sheela felt very bored one evening P. Gopal wanted to stay on for the next show Q. So they decided to go to the cinema R. They reached the theatre in time for the interval S. On the way there was a traffic jam 6. But Sheela wanted to return home (A) PSQR (B) SQPR (C) QSRP (D) SQRP 73. 1. Ferdinand rose up to receive the messenger P. At the end of his account he was moved to tears Q. He fell on his knees and thanked him R. He made him sit on a level with himself S. He listened to the circumstantial account of his voyage 6. It was a great conquest the Almighty gave to a sovereign. (A) RPSQ (B) RSPQ (C) QPRS (D) PKSQ 74. 1. In order to judge the inside of others, study your own P. and though one has one prevailing passion Q. for, men, in general are very much alike R. yet their operations are very much the same S. and another has another 6. and whatever engages or disgusts, pleases or offends you in others, will engage, disgust, please or offend others in you. (A) QPSR (B) PQRS (C) RQPS (D) PRQS 75. 1. Paucity of funds P. universities cannot make Q. essentials like books R. sufficient expenditure on S. ordains that the 6. journals and equipment. (A) QPRS (B) SPRQ (C) PQRS (D) QSRP 76. 1. The symptoms of P. and certain other changes Q. what is popularly called R. serious forgetfulness, confusion S "serility" include 6. in personality behaviour. (A) QSRP (B) PQRS (C) SRQP (D) QPSR 77. 1. When they heard the bell P. out of his clothes Q. as quickly R. every boy scrambled S. and got into bed 6. as possible. (A) QRPS (B) PSQR (C) RQSP (D) RPS.Q 78. 1. Thirty years from now P. and industry will be scarce Q. almost half of the people R. that water for drinking, farming S. then living may find 6. according to a study by Dr. S. Posten. (A) SPQR (B) SRPQ (C) QSRP (D) QSPR 79. 1. A good cry can be a P. bring relief from anxiety Q. prevent a headache or R. and it might even S. healthy way to 6. other physical consequence (A) SRQP (B) SRPQ (C) SPRQ (D) SPQR 80. 1. Long, long, time ago P. who lived with his three wives Q. in a country called Kosala R. there ruled a noble king S. and four sons 6. beautiful, graceful and well versed in all shastras. (A) PQRS (B) SPQR (C) RQPS (D) QRSP Directions: In the following passage (Question Nos. 81 to 90), some of the words have been left out. First read the passage over and try to understand what it is about. Then fill in the blanks with the help of the alternatives given. Cloze Passage Scientists have developed an electronic circuit that 81 the wiring of the human brain in some ways - an achievement that 82 revolu-tionalise computer science and 83 understanding of how nature's most powerful 84 works. The 85 built on a 86 chip the size of a finger nail, is 87 from the thinking machines of science-fiction. For one thing it cannot 88 , the way the 89 can. But researchers say it could 90 in better speech and object recognition by computers. 81. (A) imitates (B) mimics (C) limits (D) expands 82. (A) shall (B) ought (C) could (D) have 83. (A) exceed (B) improve (C) impair (D) develop 84. (A) processor (B) electronic system (C) circuit (D) brain 85. (A) computer (B) monitor (C) system (D) circuit 86. (A) silicon (B) minute (C) big (D) brown 87. (A) for (B) above (C) beyond (D) far 88. (A) equate (B) teach (C) learn (D) recognise 89. (A) computer (B) processor (C) chip (D) brain 90. (A) yield (B) give (C) respond (D) result Directions: In Question Nos. 91 to 100, you have two brief passages with five questions following each passage. Read the passages carefully and choose the best answer to each question out of the four alternatives. PASSAGE - I For any activity, discipline is the key word. It should begin with the self, then be extended to the family, neighbours, environment, workplace, society and the nation at large. It is from society that inspiration is drawn. Systems and institutions should provide the inspiration to society through performance which in turn will provide leaders, capable of rebuilding and restructuring society into a strong nation. The nationalists' spirit then becomes infectious. 91. What is the key word for Activity according to the passage? (A) Active Discipline (B) Key Discipline (C) Self Discipline (D) Discipline 92. According to the passage Discipline should begin (A) with the self (B) with the self, family and neighbours (C) with the self, family, neighbours and environment (D) with the self, family, workplace, society 93. According to the passage, where do we draw inspiration from? (A) Society (B) Society and nation (C) Environment (D) Nothing in particular 94. According to the passage, a good leader should be capable of (A) rebuilding a nation the way he/she likes (B) rebuilding and restructuring society into a strong nation (C) building a national consensus (D) rebuilding and structuring a nation 95. What is the meaning of the word "infectious" in the passage? (A) dangerous (B) spreading to everyone (C) spreading to everyone by germs (D) give disease PASSAGE -II A 23-year-old British woman was yesterday sentenced to six months in jail for leaving her two-year-old daughter home alone eight-hours-a-day, five days a week for a year while she went to work. The young mother from the central town of Warwick initially hired a babysitter, when she landed a job in a travel agency but eventually reached the point when she could no longer afford the facility, prosecutors said. The woman then started leaving the child home by herself, providing it with food and toys and removing all potentially dangerous objects from its reach. At first the mother came home at lunch time but had to stop because her daughter threw tantrums every time she left to go back to work. The mother, who was not identified, told the court, "If I had money I would not have done it. It was a case of that or not keeping my job and living on benefit". The judge, Mr. Harrison Hall, however said "Having had a child, the absolute priority is to look after it. There must be an alternative to leaving a child alone all day, a thing you would not do even to a dog". 96. The young mother had to work in the office (A) 40 hours a week (B) 8 hours a week (C) 48 hours a week (D) all the seven days a week 97. The word 'Facility' in sentence refers to (A) her job in the travel agency (B) living in a well furnished apartment (C) getting adequate salary (D) employing someone to look after the child 98- The mother stopped coming home for lunch because (A) her house was far away from the office (B) she was not able to control her angry baby (C) she had to work extra hours to earn more (D) she was not interested in looking after the baby 99. The sentence "If I had money, I would not have done it" means (A) I had money and so I did not leave the baby alone (B) I had money and so I left the baby alone (C) I had no money and so I left the baby alone (D) I had no money and so I did not leave the baby alone 100. Which one of the following statements about the judge Mr. Harrison Hall is correct? (A) He can tolerate cruelty to children but not to animals (B) He can tolerate cruelty to animals but not to children (C) He can tolerate cruelty both to children and animals (D) He can tolerate cruelty neither to children nor to animals PAPER I: PART-B GENERAL AWARENESS 101. Devaluation of currency leads to (A) expansion of export trade (B) contraction of import trade (C) expansion of import substitution (D) All of the above 102. Open market operations of RBI refer to buying and selling of (A) Commercial bills (B) Foreign exchange (C) Gold (D) Government bonds 103. Which is not the objective of Public Procurement and . Distribution system followed by Indian Government? (A) Maintain price stability through creation of buffer stocks (B) Protect the interests of both consumers and poor farmers (C) Control the production of food grains (D) Reduce personal and regional inequality in the distribution 104. Where is the Indian Institute of Foreign Trade located? (A) New Delhi (B) Hyderabad (C) Mumbai (D) Ahmedabad 105. The Centre for Agricultural Marketing is located at (A) Jaipur (B) New Delhi (C) Nagpur (D) Hyderabad 106. Per capita income is obtained by dividing National Income by (A) Total population of the country (B) Total working population (C) Area of the country (D) Volume of capital used 107. The currency of Thailand is (A) Bhat (B) Rupiah (C) Yuan (D) Yen 108. World Bank helps countries with loans for the purpose of (A) reconstruction and development (B) stimulating private investment (C) tackling foreign exchange crisis (D) meeting deficits in government budget 109. In Indian agriculture, the period from July to October-November is called (A) Rabi season (B) Kharif season (C) Pre-kharif season (D) Slack season 110. Token privatisation or deficit privatisation of public sector units occur when the government sells (A) 5% of shares (B) 10% of shares (C) 15% of shares (D) 20% of shares 111. The Narasimham Committee (1991) on financial reforms proposed for establish ment of a (A) Four tier hierarchy of the Banking structure (B) Three tier hierarchy of the Banking structure (C) Two tier hierarchy of the Banking structure (D) Unified control by the apex institutions 112. The Planning Commission of India was constituted in the year (A) 1942 (B) 1947 (C) 1950 (D) 1955 113. Who is the guardian of Fundamental Rights enumerated in Indian Constitution? (A) Supreme Court (B) Parliament (C) Constitution (D) President 114. The Sarkaria Commission was appointed to review the question of (A) centre/state relations (B) Legislative problems (C) Union territories' problems (D) Tribal areas 115. The Judges of the High Court hold office (A) during the pleasure of the Chief Justice of India (B) till they have attained 62 years of age (C) till they have attained 65 years of age (D) as long as they desire 116. The United Nations officially came into existence on (A) January 1, 1942 (B) October 3, 1944 (C) October 24, 1945 (D) June 26, 1945 117. The English Crown is an example of (A) Real executive (B) Quasi-real executive (C) Nominal executive (D) Nominated executive 118. The annual report of the UPSC is submitted to (A) The President (B) The Supreme Court (C) The Prime Minister (D) The Chairman of the Union Public Service Commission 119. In ends and means relationship, Gandhiji believed (A) Means become good if they serve the end (B) Means and ends are watertight compartments (C) Means determine the end (D) End is everything, no matter what or how the means are 120. The 'Speaker's vote' in the Lok Sabha is called (A) casting vote (B) sound vote (C) direct vote (D) indirect vote 121. Which part of the Indian Constitution deals with 'Fundamental Rights'? (A) Part I (B) Part II (C) Part III (D) Part IV 122. Which is the Greek classic that Gandhi translated into Gujarati? (A) Plato's Republic (B) Dialogues of Plato (C) Aristotle's Politics (D) Aristotle's Nicomachicm Ethics 123. To be a regional party, the minimum percentage of votes, that a party needs to secure in any election is (A) 2% (B) 3% (C) 4% (D) 5% 124. How many Fundamental Duties are in the Indian Constitution? (A) Eleven (B) Nine (C) Twenty (D) Twelve 125. Indian Penal Code came into operation in (A) 1858 (B) 1860 (C) 1859 (D) 1862 126. The First Viceroy & Governor-General of British India was (A) Lord Dalhousie (B) Sir John Lawrence (C) Warren Hastings (D) Lord Canning 127. The split between the 'Extremists' and 'Moderates' came up in the open at the Surat Congress Session in the year (A) 1905 (B) 1906 (C) 1907 (D) 1910 128. Bhulabhai Desai's most memorable achieve ment was his defence of the Indian National Army (I.N.A) personnel at the Red Fort Trial towards the end of (A) 1943 (B) 1944 (C) 1945 (D) 1946 129. St. Thomas is said to have come to India to propagate Christianity during the reign of the (A) Cheras (B) Parthians (C) Pandyas (D) Cholas 130. The First Viceroy of the Portuguese in the East was (A) Albuquerque (B) Joa de Castro (C) Francisco de Almedia (D) Nuno da Cunha 131. When was the All India Women's Con ference founded? (A) 1924 (B) 1925 (C) 1926 (D) 1927 132. The Kuka movement started in mid- Nineteenth century in (A) Western Punjab (B) Maharashtra (C) Bengal (D) MadhyaBharat 133. Who is the author of the autobiography, The Indian Struggle? (A) Annie Beasant (B) Subhas Chandra Bose (C) Chittaranjan Das (D) Sardar Vallabhbhai Patel 134. Mahatma Gandhi's remark, "A post-dated cheque on a crumbling bank" is regarding the proposals of (A) Simon Commission (B) Cripps Mission (C) Cabinet Mission (D) WavelPlan 135. Under whose leadership was the Congress Socialist Party founded in 1934? (A) Jawahailal Nehru and Mahatma Gandhi (B) Acharya Narendra Dev and Jai Prakash Narayan (C) Subhas Chandra Bose and P. C. Joshi (D) Saifuddin Kitchlew and Rajendra Prasad 136. The people of the Indus Valley Civilization usually built their houses of (A) Pucca bricks (B) Stone (C) Wood (D) All of the above 137. Which one of the following pairs of places does the National Highway number 2 join? (A) Delhi - Amritsar (B) Delhi - Mumbai (C) Delhi - Kolkata (D) Delhi - Ahmedabad 138. Watermelons grow best in (A) Alluvial soil (B) Sandy soil (C) Black soil (D) Laterite soil 139. In which state is the Maikala range situated? (A) Uttar Pradesh (B) Rajasthan (C) Bihar (D) Chhatisgarh 140. The latitude passing through the northern most part of India is (A) 35° N (B) 36° N (C) 37° N (D) 39° N 141. River erosion is at its greatest where the river's (A) depth is more (B) breadth is more (C) flow is fast (D) gradient is more 142. Nallamala hills are located in the state of (A) Orissa (B) Meghalaya (C) Andhra Pradesh (D) Gujarat 143. Excluding the East European countries and Russia, the country with the largest area under forest, is (A) USA (B) Canada (C) Australia (D) Brazil 144. In which of the following is the Great Barrier Reef located? (A) Coral Sea (B) Solomon Sea (C) Bismarck Sea (D) Arafura Sea 145. Where is the Headquarter of Zoological Survey of India located? (A) Mumbai (B) Delhi (C) Kolkata (D) Chennai 146. The South West monsoon engulfs the entire India by (A) 5th June (B) 15th June (C) 1st July (D) 15th July 147. Where was the first iron and steel industry of India established? (A) Bhadravati (B) Bhilai (C) Jamshedpur (D) Burnpur 148. Which one of the following is incorrectly matched? (A) Columbus - 1492 (B) Vasco de Gama - 1498 (C) Magellan - 1520 (D) Balboa - 1530 149. Which one of the following wavelengths of light is most effective in photosynthesis? (A) Blue (B) Green (C) Orange (D) Yellow 150. Human cloning is permitted in Britain for the purpose of (A) Reproduction (B) Research (C) Therapeutics (D) Genetics 151. Pick out the viral disease among the following (A) Hepatitis (B) Meningitis (C) Arthritis (D) Nephritis 152. Dolly, the World's First cloned animal was a (A) sheep (B) cow (C) goat (D) pig 153. The large amount of sugar present in human blood is (A) sucrose (B) glucose (C) fructose (D) lactose 154. Which one of the following is a viral disease in man? (A) Mumps (B) Plague (C) Cholera (D) Syphilis 155. The major constituent uf gobar gas, is (A) Carbon dioxide (B) Methane (C) Butane (D) Isobutane 156. The expansion for AIDS is (A) Active Immuno Deficiency Syndrome (B) Acquired Individual Disease Syndrome (C) Acquired Immuno Deficiency Syndrome (D) Acquired Immuno Disease Syndrome 157. Bee keeping is known as (A) Sericulture (B) Apiculture (C) Aquaculture (D) Agriculture 158. HYV refers to (A) Hybrid yielding variety (B) Human yellow virus (C) High yielding variety (D) Human yellow vaccine 159. Pick the odd one out based on crop season (A) Rice (B) Wheat (C) Maize (D) Cotton 160. The residue left after extracting juice from sugarbeet and sugarcane is called (A) molasses (B) bagasse (C) whey (D) biomass 161. The isotope of Uranium used in atomic reactors is (A) U235 (B) U236 (C) U237 (D) U232 162. In the process of magnetisation of a bar (A) The entire bulk of the bar gets magnetised (B) Only the surface of the bar gets magnetised (C) Only the ends of the bar get magnetised (D) Only some parts of the outer layers of the bar get magnetised 163. A falling drop of rainwater acquires the spherical shape due to (A) Viscosity (B) Surface Tension (C) Atmospheric pressure (D) Gravitational force 164. Which is the coldest among the following? (A) Mars (B) Earth (C) Pluto (D) Mercury 165. The weakest of all fundamental forces is (A) Gravitational force (B) Electrostatic force (C) Magnetic force (D) Nuclear force 166. Among the defects of eye, the shortsightedness is called (A) coma (B) hypermetropia (C) myopia (D) astigmatism 167. Which one of the following is not a radio active element? (A) Uranium (B) Thorium (C) Radium (D) Cadmium 168. Velocity of sound in air does not change with the change of (A) Temperature of air (B) Pressure of air (C) Moisture content in air (D) Wind in the direction of propagation of sound 169. The greenhouse effect is caused by the higher level of which gas in the atmosphere? (A) Carbon monoxide (B) Carbon dioxide (C) Nitrous oxide (D) Sulphur dioxide 170. Candle is a mixture of (A) Paraffin wax and stearic acid (B) Bees wax and stearic acid (C) Higher fatty acids and stearic acid (D) Bees wax and paraffin wax 171. Ethanol containing 5% water is known as (A) rectified spirit (B) denatured spirit (C) methylated alcohol (D) power alcohol 172. Brass is an alloy of copper and (A) tin (B) zinc (C) iron (D) nickel 173. Who discovered X-rays? (A) Goldstein (B) Thomson (C) Rontgen (D) Wien 174. The density of a gas is maximum at (A) Low temperature, low pressure (B) Low temperature, high pressure (C) High temperature, low pressure (D) High temperature, high pressure 175. The important ore of aluminium is (A) bauxite (B) cryolite (C) fluorspar (D) haematite 176. Aqua regia is a 1: 3 mixture, by volume, of (A) Conc. nitric acid and Conc. hydrochloric acid (B) Conc. hydrochloric acid and Conc. nitric acid (C) Conc. nitric acid and Conc. sulphuric acid (D) Conc. sulphuric acid and Conc. nitric acid 177. The maximum number of goals scored by India in the history of international hockey tournaments were in 1932 Olympics in Los Angeles when it beat USA by (A) 26-0 (B) 24-1 (C) 22-0 (D) 20-0 178. When was the Jawahar Rozgar Yojna launched? (A) 1985 (B) 1987 (C) 1989 (D) 1991 179. The cause for the Tsunami, as deduced by the seismologists, is (A) gravitational pull of the moon (B) low pressure trough in the ocean (C) deformation of sea floor and vertical displacement of water (D) sudden change in the monsoon wind 180. Eden Garden (Kolkata) is associated with (A) Basket Ball (B) Football (C) Cricket (D) Hockey 181. Which one of the following states of India records the highest sex-ratio? (A) Kerala (B) Karnataka (C) Meghalaya (D) Tamilnadu 182. Major fishing grounds are found in (A) Northern Hemisphere (B) Southern Hemisphere (C) Eastern Hemisphere (D) Western Hemisphere 183. When was the first National Forest Policy issued by the Government of India? (A) 1952 (B) 1940 (C) 1942 (D) 1999 184. Which one of the following is not a HYV of wheat? (A) Sonalika (B) Ratna (C) Kalyan Sona (D) Girija 185. What is the maximum Water Vapour content in the atmosphere? (A) 2 to 3 per cent (B) 3 to 4 per cent (C) 4 to 5 per cent (D) 5 to 6 per cent 186. Who is the first Asian to become the Head of the prestigious Trinity College of Cambridge University? (A) Amartya Sen (B) Mahendra Chaudhury (C) Nirod C. Chaudhury (D) Ketaki Mushari Dyson 187. Who gave the concept of 'Total Revolution'? (A) Jayaprakash Narayan (B) Mahatma Gandhi (C) Karl Marx (D) Lenin 188. The first mid-term elections for Lok Sabha were held in (A) 1962 (B) 1971 (C) 1977 (D) 1980 189. Who declared as his ultimate aim the wiping of every tear from every eye? (A) Jawaharlal Nehru (B) Gandhiji (C) Bal Gangadhar Tilak (D) Sardar Patel 190. Japan's Parliament is known as (A) Diet (B) Dail (C) Yuan (D) Shora 191. The convention that "once a speaker always. a speaker" is followed in (A) UK (B) USA (C) France (D) India 192. The Headquarters of the Food and Agricultural Organisation (FAO) is at (A) New York (B) Paris (C) Geneva (D) Rome 193. Dr. A. P. J. Abdul Kalam, the former Indian President, received the prestigious Bharat Ratna award in the year (A) 1992 (B) 1995 (C) 1997 (D) 1998 194. The Indian actor who played a prominent role in the Hollywood production: "The Ghost and The Darkness" is (A) OmPuri (B) Naseeruddin Shah (C) Amitabh Bachchan , (D) Anil Kapoor 195. Reserve Bank of India was nationalised in (A) 1947 (B) 1948 (C) 1949 (D) 1951 196. PSLV stands for (A) Polar Satellite Launch Vehicle (B) Polish Satellite Launch Vehicle (C) Perfect Satellite Launching Verifier (D) Preparatory Satellite Launching Vehicle 197. Indira Gandhi Cup is associated with (A) Basketball (B) Football (C) Boxing (D) Cricket 198. Who is the author of the novel 'DEVDAS'? (A) Rabindranath Tagore (B) Bankim Chandra Chattopadhyay (C) Sarat Chandra Chattopadhyay (D) Munshi Premchand 199. Nano technology deals with objects with dimensions in the range of (A) 10-3 m (B) 10-6 m (C) 10-9 m (D) 10-12 m 200. The moon is showing its same face to the earth because (A) It is not rotating about its own axis (B) Its rotation and revolution are opposite (C) Its periods of rotation and revolution are the same (D) Its rotation is faster than its revolution Source: http://entrance-exam.net/forum/general-discussion/previous-solved-papers-section-officers-excutive-assistant-council-scientific-industrial-research-books-12040.html#ixzz2E9fUtLPf UGC Paper -1 1. Feedback in communication Process __________________ (A) Explores defect in receiver (B) Clarifies the communication-Answer (C) Beneficial in understanding (D) Delights the Sender 2. Intrapersonal communication is __________________ (A) Person-to-person contact (B) Talking to oneself -Answer (C) When more than two persons are involved (D) None of the above 3. An effective communication produces in receiver ______ (A) Acceptance -Answer (B) Joy (C) Thinking (D) None of the above 4. For Company A, how much is the percent increase in production in 2000 from 1999? (A) 0.25 (B) 2.5 (C) 25 -Answer (D)12.5 5. How many units is the total production of Company A for the given years? (A) 9 crores (B) 17.75 crores (C) 12.25 crores (D) 11 crores-Answer 6. What is the difference in units produced by the two companies in 1999? (A) 1,50,000,000 -Answer (B) 15,00,00,000 (C) 15,00,000 (D) 15,000 7. How many units is the approximate average production of Company B for the given years? (A) 3 crores (B) 2.55 crores-Answer (C) 2.75 crores (D) 2.25 crores 8. In which year did both the companies have no change in production from the previous year? (A) 2000 (B) 2002 (C) 2003 (D) 2004 -Answer 9 Which of the following is not open source software? (A) Internet Explorer -Answer (B) Apache HTTP Server (C) Fedora Linux (D) Open Office 10. In MS Word, after selecting content, it can be copied by ______ (A) Shift Key + C Key (B) Alt Key + C Key (C) Enter Key (D) Ctrl Key + C Key -Answer 11 MODEM = ______________ (A) An Electric Device (B) Modulator and Demodulator-Answer (C) Internal Memory (D) None of the above 12 Multimedia is __________ (A) A computer Game (B) A software (C) A technology -Answer (D) A Media 13 What is the full form of ‘LAN’? (A) Large Area Network (B) Local Area Network-Answer (C) Little Area Network (D) None of the above 14 Look at this series: 53, 53, 40, 40, 27, 27, what number should come next? (A) 10 (B) 18 (C) 14 -Answer (D) 45 Direction (15-19) The strength of Indian Democracy lies in its tradition, in the fusion of the ideas of democracy and national independence which was the characteristic of the Indian Nationalist Movement long before independence. Although the British retained supreme authority in India until 1947, the provincial elections of 1937 provided real exercise in democratic practice before national independence. During the Pacific war India was not overrun or seriously invaded by the Japanese and after the war was over, the transfer of power to a government of the Indian Congress Party was a peaceful one as far as Britain was concerned. By 1947 'Indianisation' had already gone far in the Indian Civil Service and Army, so that the new government could start with effective instruments of central control. After independence, however, India was faced with two vast problems; the first, that of economic growth from a very low level of production and the second was that of ethnic diversity and the aspirations of sub nationalities. The Congress leadership was more aware of the former problem than of the second. As a new political elite which had rebelled not only against the British Raj but also against India's old social order, they were conscious of the need to initiate economic development and undertake social reforms, but as nationalists who had led a struggle against the alien rule on behalf of all parts of India, they took the cohesion of the Indian nation too much for granted and underestimated the centrifugal forces of ethnic division, which were bound to be accentuated rather than diminished as the popular masses were more and more drawn into politics. The Congress party was originally opposed to the idea of recognizing any division of India on a linguistic basis and preferred to retain the old provinces of British India which often cut across linguistic boundaries. However, this was later conceded as the basis for a federal 'Indian Union'. The rights granted to the States created new problems for the Central Government. The idea of making Hindi the national language of a united India was thwarted by the recalcitrance of the speakers of other important Indian languages and the autonomy of the States rendered central economic planning extremely difficult. Land reforms remained under the control of the States and many large-scale economic projects required a degree of cooperation between the Central Government and one or more of the States which, it was found, was impossible to achieve. Coordination of policies was difficult even when the Congress party was in power both in the State and at the Centre. When a Congress Government in Delhi was confronted with non-Congress parties in office in the States, it became much harder. 15. Which of the following problems was India faced with after Independence ? (A) Military attack from a country across the border. (B) Lack of coordination between the Central and State Governments. (C) Improper coordination of various Government policies (D) Increasing the production from a very low level -Answer 16. Which of the following issues was not appropriately realized by the Central Government. (A) Ethnic diversity of the people -Answer (B)A national language for the country (C) Implementation of the formulated policies (D) Centre -State relations 17. Why was central economic planning found to be difficult? (A) Multiplicity of States and Union Territories (B) Lack of coordination in different Government departments (C) Autonomy given to the States in certain matters-Answer (D) Lack of will in implementing land reforms 18. Why was the linguistic reorganization of the State accepted? (A) The States were not cooperating with the Central Government (B) Non- Congress Governments in the States demanded such a reorganization of the States (C) No common national language emerged (D) Strong pressure from the States was exerted on the Central Government tocreate such States-Answer 19. Which, according to the passage, can be cited as an exercise in democratic practice in India before Independence? (A) The handing over of power by the British to India (B) The Indianisation of the Indian Civil Service-Answer (C) A neutral role played by the Army (D) None of the above 20. The information to be collected in survey method are related to (A) Present Position (B) Aims of the research (C) The attainment of aim of research (D) All of the above-Answer 21. Research is done for (A) Knowledge of research process (B) Solving a business problem -Answer (C) Interest in research (D) Experience 22 A research problem is feasible only when (A) It is researchable (B) It has some utility (C) It is new (D) All of the above-Answer 23. One of the essential characteristics of research is (A) Sensitivity (B) Generalizability (C) Usability -Answer (D) 24. The experimental research is (A) A method for verifying a hypothesis -Answer (B) A variable controlling method (C) A method of deriving inferences (D) None of the above 25. UGC was established formally as statutory body in the year ____________ (A) 1972 (B) 1965 (C) 1956 -Answer (D) 1962 26. In India, total numbers of Open Universities are _______ (A) Fourteen -Answer (B) Sixteen (C) Eighteen (D) Fifteen 27. Who was chairman of the UGC committee 1969 appointed for administrative legislation of the universities? (A). Dr. Zakir Hussain (B) Dr. P B Gajendragadkar-Answer (C) Dr. L S Mudaliar (D) Dr. Radha Krishnan 28. UGC has launched career oriented program in (A) 1964-65 (B) 1994-95 -Answer (C) 1997-98 (D) 1980-81 29. The prime minister of India is appointed from ______ (A) The leading Party in Lok Sabha-Answer (B) The Leading Party in Rajya Sabha (C) The leading party in Lok Sabha and Rajya Sabha combined (D) None of the above 30. The study of interrelations between Organism and their environment is called___ (A) Biosphere (B) Ecology -Answer (C) Synecology (D) Autecology 31 WWF is _______ (A) World wide Fund for People (B) World welfare Fund (C) World wide Fund for Nature -Answer (D) Non of the above 32 Which was the first National Park established in India? (A) Anshi National Park (B) Gir National Park (C) Kanha National Park (D) Jim Corbett National Park-Answer 33 Fossil Fuels include (A) Oil (B) Natural Gas (C) Coal (D) All of the above-Answer 34 Noise in excess of ______ is called noise pollution (A) 40-65 db (B) 60-70 db (C) 80-100 db -Answer (D) None of the above 35 Effectiveness of teaching depends on ____ (A) Handwriting of Teacher -Answer (B) Speaking ability of Teacher (C) Qualification of the Teacher (D) Subject Understanding of the Teacher 36 Verbal Guidance is least effective in the learning of ____ (A) Aptitudes (B) Skills (C) Attitudes (D) Relationship-Answer 37 The participation of students will be maximum if ____ method is used for teaching. (A) Text Books (B) Discussion Method-Answer (C) Conference Method (D) Lectures 38 The primary responsibility of the teacher’s adjustment lies with (A) The Students (B) The Principal (C) The Community (D) The Teacher himself-Answer 39. The First Kindergarten was started by (A) William James (B) A D Clinton (C) Freidrich Forebel -Answer (D) J H Hills 40. In following questions, number series is given. One of the numbers in each series is wrong. After searching wrong number find the correct number in its place. 510, 254, 126, 64, 30, 14, 6 (A) 252 (B) 62 -Answer (C) 130 (D) 9 41. Cow : Calf :: Mother: ? (A) Daughter-Answer (B) Father (C) Brother (D) None of the above 42 Insert the missing number or letter from among the given alternatives. (A) 140 (B) 280 (C) 875 (D) 925-Answer 43. In the following question assuming the given statements to be true, find out which of the two assumptions I and II given below them is/are definitely true give answer as. (A) Only assumption I is implicit -Answer (B) Only assumption II is implicit (C) Either I or II is implicit (D) Neither I nor II is implicit (E) Both I and II are implicit Statement: The State government has decided to appoint four thousand primary school teachers during the next financial year. Assumptions: I. There are enough schools in the state to accommodate four thousand additionalprimary school teachers. II. The eligible candidates may not be interested to apply as the government may not finally appoint such a large number of primary school teachers. 44. What is the latest write-once optical storage media? (A) Digital paper (B) Magneto-optical disk (C) WORM disk (D) CD-ROM disk-Answer 45. Which of the following identifies a specific web page and its computer on the Web? (A) Web site (B) Web site address (C) URL (D) Domain Name-Answer Direction (46-47) In the following figure, rectangle, square, circle and triangle represents the regions of wheat gram, maize and rice cultivation respectively. On the basis of the figure, answer the following questions. 46. Which of the area is cultivated for wheat and maize only? (A) 8 (B) 6 (C) 5 (D) 4-Answer 47. Which of the area is cultivated for maize only? (A) 10 (B) 2 (C) 3 -Answer (D) 4 48. Pointing to a photograph. Bajpai said, “He is the son of the only daughter of the father of my brother.” How Bajpai is related to the man in the photograph? (A) Nephew (B) Brother (C) Father (D) Maternal Uncle -Answer 49. Light Year is a unit of: (A) Intensity of light (B) Distance-Answer (C) Time (D) Planetary motion 50. Tsunamis are huge sea waves caused by : (A) Earthquakes -Answer (B) Volcanoes (C) Winds (D) Icebergs UGC NET Teaching aptitude questions 1. Which one of the following is the main objective of teaching? (A) To give information related to the syllabus. (B) To develop thinking power of students.-Answer (C) To dictate notes to students. (D) To prepare students to pass the examination. 2. Which one of the following is a good method of teaching? (A) Lecture and Dictation (B) Seminar and Project-Answer (C) Seminar and Dictation (D) Dictation and Assignment 3. Teacher uses teaching aids for (A) Making teaching interesting (B) Making teaching within understanding level of students-Answer (C) Making students attentive. (D) The sake of its use. 4. Effectiveness of teaching depends on (A) Qualification of teacher (B) Personality of teacher (C) Handwriting of teacher (D) Subject understanding of teacher-Answer 5. Which of the following is not characteristic of a good question paper? (A) Objectivity (B) Subjectivity-Answer (C) No use of vague words (D) Reliable. 6. A researcher is generally expected to: (A) Study the existing literature in a field (B) Generate new principles and theories-Answer (C) Synthesize the idea given by others (D) Evaluate the findings of a study 7. One of the essential characteristics of research is: (A) Replicability (B) Generalizability (C) Usability (D) Objectivity-Answer 8. The Government of India conducts Census after every 10 years. The method of research used in this process is: (A) Case Study (B) Developmental (C) Survey-Answer (D) Experimental 9. An academic association assembled at one place to discuss the progress of its work and future plans. Such an assembly is known as a (A) Conference-Answer (B) Seminar (C) Workshop (D) Symposium 10. An investigator studied the census date for a given area and prepared a write-up based on them. Such a write-up is called (A) Research paper (B) Article-Answer (C) Thesis (D) Research report Read the following passage and answer the Question Nos. 11 to 15 The constitution guarantees every citizen the fundamental right to equality. Yet after 50 years of independence, just one perusal of the female infant mortality figures, the literacy rates and the employment opportunities for women is sufficient evidence that discrimination exists. Almost predictably, this gender, bias is evident in our political system as well. In the 13th Lok Sabha, there were only 43 women MPs out of total of 543; it is not a surprising figure, for never has women's representation in Parliament been more than 10 per cent. Historically, the manifestos of major political have always encouraged women's participation. It has been merely a charade. So, women's organizations, denied a place on merit, opted for the last resort; a reservation of seats for women in parliament and State Assemblies. Parties, which look at everything with a vote bank in mind, seemed to endorse this. Alas, this too was a mirage. But there is another aspect also. At a time when caste is the trump card, some politicians want the bill to include further quotas fro women from among minorities and backward castes. There is more to it. A survey shows that there is a general antipathy towards the bill. It is actually a classic case of doublespeak: in public, politicians were endorsing women's reservation but in the backrooms of Parliament, they were busy sabotaging it. The reasons are clear: Men just don't want to vacate their seats of power. 11. The problem raised in the passage reflects badly on our (A) Political system (B) Social behaviour-Answer (C) Individual behaviour (D) Behaviour of a group of people 12. According to the passage, political parties have mostly in mind (A) Economic prosperity (B) Vote bank-Answer (C) People' welfare (D) Patriotism 13. "Trump Card" means (A) Trying to move a dead horse (B) Playing the card cautiously (C) Sabotaging all the moves by others (D) Making the final jolt for success-Answer 14. The sentence "Men just don't want to vacate their seats of power" implies (A) Lust for power-Answer (B) Desire to serve the nation (C) Conviction in one's own political abilities (D) Political corruption 15. What is the percentage of women in the Lok Sabha (A) 10 (B) 7. 91-Answer (C) 43 (D) 9. 1 16. Informal communication network within the organization is knows as (A) Interpersonal communication (B) Intrapersonal Communication (C) Mass Communication (D) Grapevine Communication-Answer 17. TV Channel launched fro covering only Engineering and Technology subject is known as (A) Gyan Darshan (B) Vyas (C) Eklavya-Answer (D) Kisan 18. In which state the maximum number of periodicals are brought out for public information: (A) Uttar Pradesh (B) Tamil Nadu (C) Kerala-Answer (D) Punjab 19. The main objective of public broadcasting system i. e Prasar Bharti is (A) Inform, Entertainment & Education-Answer (B) Entertain, Information & Interaction (C) Educate, Interact & entertain (D) Entertainment only 20. The competerrcy of an effective communicator can be judged on the basis of: (A) Personality of communicator (B) Experience in the field (C) Interactivity with target audience (D) Meeting the needs of target audience.-Answer 21. Which one of the following belongs to the category of homogeneous date: (A) Multi-storeyed houses in a colony-Answer (B) Trees in a garden (C) Vehicular traffic on a highway (D) Student population in a class 22. In which of the following ways a theory is not different from a belief? (A) Antecedent - consequent (B) Acceptability-Answer (C) Verifiability (D) Demonstratability 23. The state - "Honesty is the best policy" is (A) A fact (B) An value (C) An opinion (D) A value judgement-Answer 24. Which one is like pillar, pole and standard? (A) Beam-Answer (B) Plank (C) Shaft (D) Timber 25. Following incomplete series is presented. Find out the number which should come at the place of question mark which will complete the series: 4, 16, 36, 64, ? (A) 300-Answer (B) 200 (C) 100 (D) 150 26. The following question is based on the diagram given below. If the two big circles represent animals living on soil and those living in water, and the small circle stands for the animals who both live on soil and in water, which figure represents the relationships among them. D--Answer 27. Of the following statement, there are two statements both of which cannot be true but both can be false. Which are these two statements? (i) All machines make noise (ii) Some machines are noisy (iii) No machine makes noise (iv) Some machines are not noisy (A) (i) and (ii) (B) (iii) and (iv) (C) (i) and (iii) (D) (ii) and (iv)-Answer 28. In the following question a statement is followed by two assumptions. (i) and (ii) . An assumption is something supposed or taken for granted. Consider the statement and the following assumptions and decide which of the following assumptions is implicit in the statement. Statement: We need not worry about errors but must try to learn from our errors. Assumptions: (i) Errors may take place when we are carrying out certain work. (ii) We are capable of benefiting from the past and improve our chances of error-free work. (A) Only assumption (i) is implicit (B) Only assumption (ii) is implicit (C) Either assumption (i) or (ii) is implicit (D) Both the assumptions are implicit-Answer 29. The question below is followed by two arguments numbered (i) and (ii) Decide which of the arguments is 'strong' and which is 'weak'. Choose the correct answer from the given below Should the press exercise some self-restraint? (i) Yes, they should not publish new items which may incite the readers to indulge in wrong practices. (ii) No. it is the responsibility of the press to present the truth irrespective of the consequences. (A) Only the argument (i) is strong-Answer (B) Only the argument (ii) is strong (C) Neither argument (i) nor (ii) is strong (D) Both the arguments (i) and (ii) are strong 30. Study the argument and the inference drawn from that argument. Given below carefully. Argument: Anything that goes up definitely falls down. Helicopter goes up. Inference: So the helicopter will definitely fall down. What in your opinion is the inference drawn from the argument? (A) Valid (B) Invalid (C) Doubtful (D) Long drawn one-Answer Four students W, X, Y, Z appeared in four papers, I, II, III and IV in a test. Their scores out of 100 are given below. Students Papers I II III IV W 60 81 45 55 X 59 43 51 A Y 74 A 71 65 Z 72 76 A 68 Where 'A' stands for absent Where 'A' stands for absent Read the above table and answer below mentioned Questions 31 to 35 31. Which candidate has secured between 60-65% marks in aggregate (A) W-Answer (B) X (C) Y (D) Z 32. Who has obtained the lowest average in aggregate. (A) W (B) X-Answer (C) Y (D) Z 33. Who has obtained the highest average (A) W-Answer (B) X (C) Y (D) Z 34. In which paper the lowest marks were obtained by thecandiates (A) I (B) II-Answer (C) III (D) IV 35. Which candidate has secured the highest percentage in the papers appeared (A) W (B) X (C) Y (D) Z-Answer 36. ICT stands for (A) Information common technology (B) Information & communication technology-Answer (C) Information and computer technology (D) Inter connected technology 37. Computer Can (A) Process both quantitative and qualitative information (B) Store huge information (C) Process information and fast accurately (D) All the above.-Answer 38. Satellite Communication works through (A) Rader (B) Transponder-Answer (C) Receptor (D) Transmitter 39. A Computer is that machine which works more like a human brain. This definition of computer is (A) Correct-Answer (B) Incorrect (C) Partially correct (D) None of the above. 40. Information and communication technology includes (A) E-mail (B) Internet (C) Education television (D) All the above.-Answer 41. It is believed that our globe is warming progressively. This global warming will eventually result in. (A) Increase in availability of usable land. (B) Uniformity of climate at equator and poles. (C) Fall in the sea level (D) melting of polar ice.-Answer 42. In which parts of India ground water is affected with arsenic contamination? (A) Haryana (B) Andhra Pradesh (C) Sikkim (D) West Bengal-Answer 43. Sunderban in Hooghly delta is known for (A) Grasslands (B) Conifers (C) Mangroves-Answer (D) Arid forests 44. Sardar Sarover dam is located on the river (A) Ganga (B) Godavari (C) Mahanadi (D) Narmada-Answer 45. Which one of the following trees has medicinal value? (A) Pine (B) Teak (C) Neem-Answer (D) Oak 46. Which one of the following is not considered a part of technical education in India: (A) Medical-Answer (B) Management (C) Pharmaceutical (D) Aeronautical 47. Which of the following is a Central university (A) Mumbai University (B) Calcutta University (C) Delhi University-Answer (D) Madras University 48. Identify the main Principle on which the Parliamentary System Operates (A) Responsibility of Executive to Legislature-Answer (B) Supremacy of Parliament (C) Supremacy of Judiciary (D) Theory of Separation of Power 49. The reservation of seats for women in the Panchayat Raj Institutions is: (A) 30 % of the total seats (B) 33 % of the total seate-Answer (C) 33% of the total population (D) In Proportion to their population 50. Match list I with list II and select the correct answer from the code given below: LIST ( Institutions) LIST II( Locations) 1. Indian Veterinary Research Institute (i) Pune 2. Institute of Armament Technology (ii) Izat Nagar 3. Indian Institute of Science (iii) Delhi 4. National Institute for Educational Pannesi (iv) Bangalore and Administrators (A) 1(ii), 2(i), 3(iv), 4(iii)-Answer (B) 1(ii), 2(iv), 3(ii), 4(iii) (C) 1(ii), 2(iii), 3(i), 4(iv) (D) 1(iv), 2(iii), 3(ii), 4(i) CSIR CASE UGC NET -2013 Engineering science questions with answers,CSIR UGC NET -2013 PAPER ? II CSIR UGC NET previous years solved question papers with answers and detailed explanations,CSIR UGC NET free solved sample placement papers 1. The output of a logic gate is 1 when all its inputs are at logic 0. The gate is either (A) A NAND or an EX-OR (B) An OR or an EX-NOR (C) An AND or an EX-OR (D) A NOR or an EX-NOR (Ans) Hints and Solution : The output of a logic gate is 1 when all inputs are at logic 0. The gate is either a NOR or an EX- NOR. Input Output A B Y 0 0 1 0 1 0 1 0 0 1 1 0 Truth Table for NOR Gate Input Output A B Y 0 0 1 0 1 0 1 0 0 1 1 1 Truth Table for EX-NOR Gate 2. What is the simplification of the following Boolean expression in a Product-Of-Sum form? (A) Out = (A+B+ NOT(C)) (A+NOT(C)+ D) (C+NOT(D)) (B) Out = (A+B) (A+NOT(D)) (B+NOT(C)+D) (C) Out = (B+C+ NOT(D)) (A+C+ NOT(D)) ( NOT(C)+D) (Ans) (D) Out = (A+NOT(B)+D) (C+D) (B+NOT(C)+D) Hints and Solution : Transfer the seven maxterms to the map below as 0s. Map the 0s as they appear left to right top to bottom on the map above. Now form groups of cells. The final result is product of the three sums i.e. Out = (B+C+ NOT(D)) (A+C+ NOT(D)) ( NOT(C)+D) 3. What is the appropriate form for the given k-map? (A) F(W,X,Y,Z) = ?m (0,3,4,6,8,10,11,12,14) (B) F(W,X,Y,Z) = ?m (0,2,5,6,8,10,13,14,15) (Ans) (C) F(W,X,Y,Z) = ?m (1,2,5,6,8,9,11,14,15) (D) F(W,X,Y,Z) = ?m (1,2,3,7,9,10,11,14,15) F(W,X,Y,Z) = ?m (0,2,5,6,8,10,13,14,15) 4. How many fibres are required by a unidirectional and bidirectional ring respectively,to support their working traffic? (A) 1 and 1 (B) 2 and 1 (C) 2 and 2 (D) 1 and 2 (Ans) Hints and Solution : SONET rings can be classified by the routing principle and the SONET overhead used for triggering protection switching. A ring is called a unidirectional ring if bidirectional working signals follow opposite physical routes around a ring , while bidirectional working signals in a bidirectional ring follow the same route. Due to this routing principle, a unidirectional and a bidirectional ring, require one and two fibers respectively, to support their working traffic. 5. Find odd one out related to transmission media cables? (A) Basic rate ISDN can transmit data at a rate of 512 kilobits per second on an existing local telephone line. (Ans) (B) A T1 line is a dedicated telephone connection of 24 channels. (C) A T1 channel can be configured to carry either voice or data traffic. (D) Cable modems provide high-speed transmission over cable TV lines and are shared by many users. Hints and Solution : Each ISDN line is made up of separate 64-Kbps "channels" for sending and receiving calls, plus a channel that is used primarily for sign Numerical Ability Model Questions UGC LDC clerk examination model questions with answers-Aptitude, analytical, reasoning, general intelligence, general awareness, english questions with answers Directions (Q.1.5) In each of the following questions a number series is given which has only one wrong number. You have to find out the wrong number. 1. 7.5 47.5 87.5 157.5 247.5 357.5 487.5 (1) 357.5 (2) 87.5 (3) 157.5 (4) 7.5 (5) 47.5 (Ans) 2. 1500 1581 1664 1749 1833 1925 2016 (1) 1581 (2) 1664 (3) 1833 (Ans) (4) 1925 (5) 1749 3. 1331 2197 3375 4914 6859 9261 12167 (1) 4914 (Ans) (2) 6859 (3) 9261 (4) 2197 (5) 12167 4. 13 16 21 27 39 52 69 (1) 21 (2) 39 (3) 27 (Ans) (4) 52 (5) 16 5. 66 91 120 153 190 233 276 (1) 120 (2) 233 (Ans) (3) 153 (4) 276 (5) 190 Directions (Q.6.-10) What will come in place of the question marks (?) in the following questions? 6. 22.5 ? 0.05 = ? (1) 11.25 (2) 1.125 (Ans) (3) 22.55 (4) 112.5 (5) None of these 7. 999 + 111 ? 0.5 = ? (1) 555 (2) 500 (3) 1054.5 (Ans) (4) 1110.5 (5) None of these 8. 40% of 250 = 50% of ? (1) 200 (Ans) (2) 100 (3) 150 (4) 400 (5) None of these 9 (4)? = 1024 (1) 1 (2) 2 (3) 3 (4) 4 (5) None of these (Ans) 10. ½ +¼ + ¾ + 2/3 = ? (1) 2 1/5 (2) 1/16 (3) 2 1/16 (4) 2 1/6 (Ans) (5) None of these Directions (Q.11.18) What approximate value should come in place of the question mark (?) in the following questions ? (You are not expected to calculate the exact value.) 11. (98 ? 198) ÷ (48 ? 148) = ? (1) 3.13 (2) 2.43 (3) 2.73 (Ans) (4) 3.53 (5) 2.41 12. (2.0001)3 =? (1) 6 (2) 6.75 (3) 5 (4) 4 (5) 8 (Ans) 13. 5.6% of 240 ÷ 0.3% of 480 = ? (1) 8.43 (2) 9.33 (Ans) (3) 7.53 (4) 8.33 (5) 13.44 14. ?4556 + ?7943 ? 56 =? (1) 100 (Ans) (2) 150 (3) 200 (4) 250 (5) 125 15. (1786 + 23.894 + 3251) ÷ 50 = ? (1) 81 (2) 121 (3) 131 (4) 101 (Ans) (5) 91 16. The difference between 55% of a number and 25% of the same number is 11.10. What is 75% of that number? (1) 27.75 (Ans) (2) 37 (3) 21.25 (4) 45 (5) None of these 17. The average of the ages of 3 friends is 23. Even if the age of the 4th friend is added the average remains 23. What is the age of the 4th friend ? (1) 32 (2) 21 (3) 23 (Ans) (4) Cannot be determined (5) None of these 18. 8 men working for 9 hours a day complete a piece of work in 20 days. In how many days can 7 men working for 10 hours a day complete the same piece of work? (1) 21 days (2) 20 3/5 days (3) 20 ½ days (4) 21 3/7 days (5) None of these (Ans) 19. Samir drove at the speed of 45 km from home to a resort. Returning over the same route, he got stuck in traffic and took an hour longer, also he could drive only at the speed of 40 km. How many kilometers did he drive each way ? (1) 250 km (2) 300 km (3) 310 km (4) 275 km (5) None of these (Ans) 20. What is the greater of two numbers whose product is 640, if the sum of the two numbers, exceeds their difference by 32? (1) 45 (2) 50 (3) 55 (4) 40 (Ans) (5) None of these